Local Anesthetics Flashcards

1
Q

A 36-year-old woman undergoes reduction mammaplasty. An intravenous infusion of vancomycin 1.5 g is started preoperatively as antibiotic prophylaxis. Medical history includes allergies to penicillin and clindamycin, but the patient has never received vancomycin. After 5 minutes of continuous infusion, an erythematous rash develops over the patient’s torso, neck, and face, and she reports headache and generalized pruritus. She remains hemodynamically stable and is otherwise asymptomatic. After discontinuing the antibiotic infusion, which of the following is the most appropriate next step in management?

A) Administer epinephrine intramuscularly
B) Administer morphine intravenously
C) Document the patients allergy to vancomycin in the medical record
D) Order a STAT vancomycin serum level
E) Resume the infusion at a slower rate once the patients symptoms improve

A

The correct response is Option E.

This patient developed a mild form of red man syndrome (RMS), caused by vancomycin administration. The most appropriate next step in management after discontinuing the antibiotic infusion is symptomatic treatment with an antihistamine drug (eg, intravenous or oral administration of diphenhydramine). The vancomycin infusion can be resumed after resolution of the rash and other symptoms, albeit at a slower rate.

Vancomycin can cause a plethora of hypersensitivity reactions, ranging from localized skin reactions to anaphylactic shock. RMS is its most common form and consists of an erythematous rash that involves the face, neck, and upper torso, commonly associated with generalized discomfort and cutaneous pruritus or burning. Patients can experience headache, chills, fever, perioral paresthesia, and/or dizziness. Although rare, chest pain, diffuse muscle spasms, angioedema, and hypotension can also occur. RMS is associated with rapid intravenous infusion of vancomycin, occurring in less than 10% of patients with serious infections who receive 1 g infused over 1 hour. However, healthy patients may have a higher susceptibility. Contrary to true anaphylaxis, RMS is not thought to be mediated by vancomycin-specific antibodies, and can occur even with its first administration.

Intramuscular injection of epinephrine is indicated for anaphylactic reactions. Morphine and other opioids have been shown to increase the rate of RMS in patients receiving vancomycin infusion, and, therefore, should be avoided. A vancomycin serum level at this time would unlikely provide any valuable information. RMS is not considered a patient-specific allergy, but rather an idiosyncratic reaction related to speed of drug infusion.

2018

How well did you know this?
1
Not at all
2
3
4
5
Perfectly
2
Q

A 45-year-old man is brought to the emergency department after sustaining a gunshot wound to the chest. He is in respiratory distress and has decreased breath sounds on the right side. Needle decompression is performed in the right chest, and a tube thoracostomy is placed. The patient stabilizes. Which of the following initial findings is most likely in this patient?

A) Cardiac murmur
B) Decreased urine output
C) Hyperresonance of the left chest
D) Hypertension
E) Tracheal deviation to the left

A

The correct response is Option E.

Tension pneumothorax occurs when there is injury to the lung and air leakage into the pleural space that cannot escape. Each subsequent breath increases the volume of air in the pleural space and increases the intrapleural pressure. As pressure builds, structures of the chest are compressed, impairing ventilation and decreasing venous return.

Tension pneumothorax is a clinical diagnosis. Features include tachypnea, dyspnea, jugular venous distention, decreased air entry, hyperresonance on the affected side, tracheal deviation to the opposite side, and hypotension. Treatment is immediate decompression of the pleural space with a long, large-bore needle followed by insertion of a chest tube.

In this patient with a history of penetrating chest trauma, decreased breath sounds, and respiratory distress, tension pneumothorax is diagnosed. The trachea will be deviated to the contralateral side (left) and the patient will be hyperresonant to percussion on the right side. Cardiac murmur and hypertension are not features of tension pneumothorax. Urine output is irrelevant.

2018

How well did you know this?
1
Not at all
2
3
4
5
Perfectly
3
Q

A 30-year-old man comes to the emergency department 4 months after undergoing transplantation of the right hand because he has a painful, burning rash on the transplanted hand. Physical examination of the hand shows a maculopapular erythematous rash. Which of the following types of rejection is most likely in this patient?

A) Hyperacute
B) Accelerated
C) Acute
D) Chronic
E) Graft-versus-host

A

The correct response is Option C.

In transplantation, rejection is categorized as hyperacute, accelerated, acute, and chronic.

Hyperacute rejection is a humoral response mediated by antibodies that are already present in the host at the time of transplantation. It starts in the operating room, at the time blood flow to the transplanted organ is reestablished. Exposure to the graft tissue activates preexistent antibodies, which then activate the complement system, causing diffuse thrombosis in the graft’s microcirculation and rapid ischemic failure, usually within the first few minutes to hours of graft reperfusion.

Accelerated rejection is a cellular and humoral response that usually occurs between the second and fifth day after transplantation. Considered a variant of hyperacute rejection, it also results from recipient presensitization to donor tissue antigens, but without the formation of large numbers of preexistent antibodies. Instead, memory cells mediate a rapid immune response against the graft. Complement activation and intravascular thrombosis usually leads to graft loss.

This patient is experiencing acute rejection, which is regulated by the activation of T-cells. It can occur at any time after the fifth postoperative day, although it is more common within the first 4 to 6 months. Clinical presentation depends on the transplanted organ, but is usually characterized by short-term organ dysfunction (eg, liver, kidney) or graft cutaneous/mucosal manifestations. A graft biopsy is usually necessary for diagnosis. The majority of acute rejections respond well to higher levels of immunosuppressive therapy, although recurrent events might be predictive of the development of chronic rejection.

Chronic rejection is both an antibody- and cell-mediated immune response that causes indolent, progressive arterial sclerosis and fibrosis of the transplanted organ (eg, coronary atherosclerosis in hearts, bronchiolitis obliterans in lungs, vanishing bile duct syndrome in livers, etc). It usually clinically manifests itself several months to years after transplantation. Initial measurable organ dysfunction usually progresses to failure.

Graft-versus-host disease is a cellular response caused by activation of the transplanted graft’s immune cells by the recipient’s tissues. It most commonly occurs several months after bone marrow or stem cell transplantation. Clinical manifestation varies, but usually involves tissues/organs of the recipient individual.

2018

How well did you know this?
1
Not at all
2
3
4
5
Perfectly
4
Q

A 45-year-old man is brought to the emergency department/intensive care unit after sustaining multiple facial fractures and a closed head injury with intraparenchymal hemorrhage in a motor vehicle collision. He is currently intubated and unresponsive. Serum sodium concentration is 140 mEq/L. Which of the following is this patient’s free water deficit?

A) 0 L
B) 1 L
C) 2 L
D) 3 L
E) 4 L

A

The correct response is Option A.

Free water deficit is calculated by the formula: Water deficit = normal body water x (1-(Serum Na/140)). The calculated free water deficit is the amount of free water required to bring the sodium concentration back to normal. Free water deficits and the associated hypernatremia can occur with diabetes insipidus after closed head injuries and need to be corrected if present. Although this patient does have a closed head injury and is unresponsive, his serum sodium concentration is in the normal range, and no calculated free water replacement is necessary.

2018

How well did you know this?
1
Not at all
2
3
4
5
Perfectly
5
Q

Temporary paresthesia is most likely in which of the following teeth after an infraorbital nerve block is performed?

A) Both central incisors, ipsilateral lateral incisor and canine
B) Both central incisors, ipsilateral lateral incisor, canine, and first and second molars
C) Ipsilateral central incisor, lateral incisor, canine, and both bicuspids
D) Ipsilateral lateral incisor, canine, both bicuspids, and first and second molars
E) Ipsilateral canine, both bicuspids, and first and second molars

A

The correct response is Option C.

Anesthetic blocks are routinely performed for office procedures. They are especially useful for facial procedures because of their reliable anatomy. Understanding the anatomy of each of the branches of the trigeminal nerve is important to successfully use this technique. The superior alveolar branch is a branch of the infraorbital nerve that controls sensation to the central and lateral incisor, the canine, and both bicuspid teeth.

The first and second molars are not innervated by branches of the infraorbital nerve. The contralateral central incisor is not innervated by the contralateral infraorbital nerve.

2018

How well did you know this?
1
Not at all
2
3
4
5
Perfectly
6
Q

Which of the following patient positioning interventions is most appropriate for prevention of perioperative peripheral neuropathy?

A) Abducting the arms at 100 degrees in supine patients
B) Pronating the forearms in supine patients with arms tucked at side
C) Securing the arms in dorsal extension in supine patients
D) Stabilizing the neck in extended position in prone patients
E) Using leg holders with heel support in lithotomy position

A

The correct response is Option E.

For patients in lithotomy position, the use of leg holders that incorporate heel support may prevent compression injury to the peroneal nerve. Contrarily, leg holders without heel support or the use of wraps to maintain the leg on leg rests may increase the risk of compression injuries to this nerve. For prevention of brachial plexus perioperative neuropathy in the supine patient, arm abduction should be limited to no greater than 90 degrees. In addition, dorsal extension of the arm (poorly padded or sagging arm boards) should be avoided.

For prevention of ulnar perioperative neuropathy in the supine patient, care should be taken to decrease pressure on the post-condylar (ulnar) groove of the humerus. When the upper extremity is on an arm board, the forearm should be positioned in either supination or neutral positions. When tucked at side, the forearm should be in a neutral position.

When in prone position, the patient’s neck should be well stabilized in neutral, nonextended position. Although neck extension has not been reportedly associated with peripheral neuropathies, authors have reported cases of vertebral artery injury.

2018

How well did you know this?
1
Not at all
2
3
4
5
Perfectly
7
Q

A 28-year-old woman, gravida 2, para 1, is brought to the emergency department after a motor vehicle collision in which she was a restrained passenger. ATLS evaluation shows that her vital signs are stable, and she is calm and lucid. The patient reports that she is 32 weeks’ pregnant. Repeat blood pressure is 80/60 mmHg, and the patient reports that she feels light-headed; tachycardia is noted. After maintaining cervical spine precautions, which of the following is the most appropriate initial management of this patient?

A) Administer 2 L crystalloid bolus
B) Intubate the patient
C) Logroll the patient onto her left side
D) Place patient in Trendelenburg position
E) Transfuse 2 units of blood

A

The correct response is Option C.

Pregnancy alters normal physiology as well as anatomy. At 32 weeks’ pregnancy, gravid uterus and fetus is anatomically at the costal margin and can be somewhat protective of bowel injury in a restrained passenger. A hematocrit of 31 to 35% is within normal range; however, the overall volume has increased dramatically and a loss of 1.2 to 1.5 L is well tolerated without symptoms.

Another crucial anatomical difference is the gravid uterus and its ability to compress the vena cava while in the supine position. Thirty percent decreased cardiac output can occur from the uterus occluding venous return from the lower extremity. In order to avoid this, the uterus should be physically displaced to the patient’s left side to allow return from the inferior vena cava. Should there be cervical spine precautions, the board can be rolled to the left 4 to 6 inches, thus alleviating pressure. In general, pregnant patients should be monitored on their left side.

Maternal well-being is the most crucial for fetal well-being, as the number one cause of fetal death is maternal hypotension and maternal death. The second most common cause of fetal death is abruption of the placenta. Fetal monitoring is extremely important, especially after 20 to 24 weeks’ gestation. However, in this case there is no evidence of cramping or tender abdomen.

Advanced Trauma Life Support (ATLS) protocol dictates airway, breathing, and circulation are normal, and, in this case, her airway and breathing are normal. The most appropriate next step is reestablishing this patient’s circulation, which is alleviating the initial possibility of decreased venous return. Should this not occur, then following the protocol for crystalloid resuscitation would be the most appropriate.

2018

How well did you know this?
1
Not at all
2
3
4
5
Perfectly
8
Q

A 42-year-old woman who is BRCA2-positive undergoes delayed reconstruction of both breasts with deep inferior epigastric artery perforator (DIEP) flaps. Medical history includes prophylactic mastectomy and failed implant reconstruction 3 months ago. After closure of the fascia of the rectus abdominis muscles, the triangle bounded by the latissimus dorsi, external oblique, and iliac crest is identified on each side. Infiltration of bupivacaine into which of the following planes is most likely to provide postoperative pain control of the abdominal flap donor site in this patient?

A) Between the external oblique and the internal oblique
B) Between the internal oblique and the transversus abdominis
C) Between the transversalis fascia and the peritoneum
D) Between the transversus abdominis and the transversalis fascia
E) In the deep subcutaneous tissues, superficial to the external oblique

A

The correct response is Option B.

The peripheral nerve block described is a transversus abdominis plane, or TAP, block. The lumbar (Petit) triangle, which is located at the lateral extent of the abdomen, is formed by the borders of the latissimus dorsi muscle posteriorly, the external oblique anteriorly, and the iliac crest inferiorly. Furthermore, the location can be estimated by the intersection of a horizontal line through the umbilicus and the vertical anterior axillary line. The intercostal nerves providing sensation to the anterior abdominal wall from T6 to L1 are located in the plane between the internal oblique and the transversus abdominis muscles. Long-acting local anesthetic infiltration in this plane is generally approached through the lumbar triangle. Multiple techniques have been described. These include visualized injection with the assistance of ultrasound guidance, catheter insertion under direct vision with surgical cutdown to the appropriate plane, and blind injection using a blunt needle, feeling for the two punctures of the external and internal obliques, respectively. Multiple studies have shown improved postoperative pain control in the abdominal donor site with this technique in breast reconstruction, and patients have demonstrated marked decreases in opioid consumption as well. The other options would be inappropriate for nerve block as the intercostal nerves are not found in those tissue planes.

2018

How well did you know this?
1
Not at all
2
3
4
5
Perfectly
9
Q

A 32-year-old man, who is a football player, is brought to the emergency department after being found unconscious and facedown in his home. It is estimated that he was in that position for 6 hours. Physical examination shows swelling of the right forearm and hand. Which of the following physiologic abnormalities is most likely in this patient?

A) Hyperkalemia, hypercalcemia, metabolic acidosis
B) Hyperkalemia, hypocalcemia, metabolic acidosis
C) Hypokalemia, hypercalcemia, metabolic alkalosis
D) Hypokalemia, hypocalcemia, metabolic acidosis
E) Hypokalemia, hypocalcemia, metabolic alkalosis

A

The correct response is Option B.

The physiological abnormalities that result from rhabdomyolysis are caused because of crush injury to the muscle. The crush injury causes pressure or stretching of the muscle and sarcolemmal membrane. As the sarcolemmal membrane is stretched, sodium, calcium (hypocalcemia), and water leak into the sarcoplasm, trapping extracellular fluid inside muscle cells. In addition to the influx of these elements into the cell, the cell releases potassium (hyperkalemia) and other toxic substances such as myoglobin, phosphate, and urate into the circulatory system (metabolic acidosis). The end result of these events is shock, hyperkalemia, hypocalcemia, metabolic acidosis, compartment syndrome, and acute renal failure. Acute renal failure results because of a combination of hypovolemia with subsequent renal vasoconstriction, metabolic acidosis and the influx of nephrotoxic substances such as myoglobin, urate, and phosphate.

2018

How well did you know this?
1
Not at all
2
3
4
5
Perfectly
10
Q

A 56-year-old woman has been in the intensive care unit since she was struck by a car 2 weeks ago. She sustained multiple facial fractures, severe closed head injury with cerebral contusion, pelvic fractures, and multiple lower extremity fractures. The patient required emergent intubation and has been in a coma since the injury. The patient’s current serum sodium concentration is 148 mEq/L, and urinalysis demonstrates dilute urine with low osmolality. Neurogenic diabetes insipidus is suspected. A deficiency in which of the following hormones is most likely the principal cause of this electrolyte imbalance?

A) Aldosterone
B) Angiotensin
C) Cortisol
D) Renin
E) Vasopressin

A

The correct response is Option E.

Neurogenic or central diabetes insipidus is often caused by head trauma resulting in a lack of vasopressin production. Vasopressin is released by the posterior pituitary gland but, unlike other pituitary hormones, is produced in the hypothalamus. Because of this unique relationship, traumatically induced neurogenic diabetes insipidus may occur due to damage to the hypothalamus, pituitary stalk, or posterior pituitary. Vasopressin, also known as antidiuretic hormone (ADH), plays a key role in fluid homeostasis by affecting water retention in the kidneys at the collecting ducts and distal convoluted tubule.

Cortisol is a hormone secreted by the adrenal glands that plays a role in numerous functions including maintaining normal metabolism and regulation of fluid balance. Angiotensin and aldosterone are key hormones of the renin-angiotensin system that is involved in the regulation of plasma sodium concentration and arterial blood pressure. Renin (also known as angiotensinogenase) is an enzyme that converts angiotensinogen to angiotensin. Angiotensin is a peptide hormone that causes vasoconstriction. It is formed by the action of renin on the liver-derived precursor, angiotensinogen, followed by the action of angiotensin-converting enzyme (ACE) to convert angiotensin I to the active form angiotensin II. Aldosterone is a steroid hormone produced by the adrenal glands and released in response to increased angiotensin. It plays a central role in regulation of plasma sodium, extracellular potassium, and arterial blood pressure. Cortisol, renin, angiotensin, and aldosterone deficiencies are not created by neurogenic diabetes insipidus related to head trauma.

2018

How well did you know this?
1
Not at all
2
3
4
5
Perfectly
11
Q

An otherwise healthy 154-lb (70-kg) woman undergoes a planned facial aesthetic procedure using conscious sedation. The anesthetic plan includes a combination of intravenous 0.25 mg/kg midazolam and 12.5 mg/kg fentanyl. Midway through the procedure, two consecutive noninvasive blood pressure readings taken 5 minutes apart demonstrate a systolic blood pressure of 85 mmHg. Which of the following is the most appropriate next step in management of this patient’s hypotension?

A) Administer flumazenil
B) Administer naloxone
C) Administer a vasopressor intravenous bolus (ephedrine or phenylephrine)
D) Begin cardiopulmonary resuscitation
E) Increase intravenous fluids

A

The correct response is Option E.

Conscious sedation is a common anesthetic technique used routinely by plastic surgeons for cosmetic procedures. The patient described is healthy and of average size. The two drugs used in this study are the most commonly used agents for this scenario. The dose chosen is on the low end of normal for both drugs. Increasing the amount of intravenous fluids is the correct answer because in the Marcus et al paper, 20% of the patients who underwent a similar conscious sedation procedure experienced this scenario. One hundred percent of these patients responded appropriately to increased intravenous fluid. Administration of flumazenil to reverse midazolam toxicity is unnecessary as the patient is not experiencing midazolam toxicity. The dosage of midazolam is within the normal range. The patient is not demonstrating any additional signs or symptoms of narcotic (fentanyl) toxicity. The dose is on the low side of normal. Reversal of fentanyl is not indicated. The patient’s systolic blood pressure is only mildly low with no other symptoms in an otherwise healthy adult woman. Vasopressors would only be indicated if a fluid challenge was unsuccessful.

2018

How well did you know this?
1
Not at all
2
3
4
5
Perfectly
12
Q

A 60-year-old man is transferred to the surgical intensive care unit (SICU) following ventral hernia repair. Medical history includes coronary artery disease. Shortly after arrival in the SICU, the 12-lead ECG shown is obtained. This finding is most consistent with which of the following cardiac dysrhythmias?

A) Atrial fibrillation
B) Atrial flutter
C) First-degree AV block
D) Multifocal atrial tachycardia
E) Second-degree AV block

A

The correct response is Option E.

Cardiac dysrhythmias in the postoperative setting are extremely common, occurring 9% of the time in non-cardiac surgery patients without a history of cardiac disease to over 40% in cardiac surgery patients. In the surgical intensive care unit (SICU) setting, cardiac dysrhythmias may have several causes including hypoxia, cardiac ischemia, catecholamine excess, routine medications, and electrolyte abnormalities. It is important to be able to identify the characteristic electrocardiographic features of common dysrhythmias.

The PR interval is a measure of the conduction time through the AV node and bundle of His. When the PR interval is prolonged, the patient has a first-degree AV block. Second degree AV block occurs with intermittent failure of the conduction of the impulse to the ventricles. Progressive prolongation of the PR interval until a failure of conduction (“dropped” QRS complex) occurs is second degree AV block (Wenckebach block).

Multifocal atrial tachycardia is thought to be caused by abnormal automaticity. It demonstrates an irregular rate and rhythm, characterized by the presence of three or more morphologically different P waves on EKG with increased rates of 110 to 140 beats per minute.

Atrial fibrillation is one of the most common dysrhythmias encountered in an ICU setting. It is associated with dilation of the left atrium due to fluid shifts or structural heart disease. Echocardiographically, it is characterized by normal complex tachycardia without P waves.

Atrial flutter is associated with atrial enlargement and is caused by a reentrant circuit within atrial tissue. Increased atrial rates of 250 to 350 beats per minute are common, and not all of the atrial beats pass to the ventricle, typically in a 2:1 ratio. The EKG has a characteristic “sawtooth” flutter wave.

2018

How well did you know this?
1
Not at all
2
3
4
5
Perfectly
13
Q

Compared with the code team, which of the following is a typical criterion for calling the Rapid Response Team (RRT)?

A) Absence of respiratory effort
B) No recordable blood pressure
C) No recordable pulse
D) Rapid heart rate
E) Unresponsiveness

A

The correct response is Option D.

The Rapid Response Team (RRT) was conceptualized to intervene in the care of a greater number of hospitalized patients at an earlier stage of clinical deterioration. The goal of the RRT is to act acutely to prevent catastrophic adverse events.

The RRT would typically be called for hypotension, rapid heart rate, respiratory distress, and altered consciousness. While the traditional code team would treat cardiac arrest, respiratory arrest, and airway obstruction, the RRT would assess and manage sepsis, pulmonary edema, arrhythmia and respiratory failure.

Typically, hospital mortality in patients treated is 70 to 90% for the traditional code team, while it is 0 to 20% for the RRT.

2018

How well did you know this?
1
Not at all
2
3
4
5
Perfectly
14
Q

A 35-year-old woman is scheduled for abdominoplasty with flank liposuction. Regional anesthetic block is planned. The most appropriate location for placement of the anesthetic is between which of the following?

A) External oblique muscle and internal oblique muscle
B) Internal oblique muscle and transversus abdominis muscle
C) Skin and external oblique muscle
D) Transversalis fascia and peritoneum
E) Transversus abdominis muscle and transversalis fascia

A

The correct response is Option B.

The transversus abdominis plane (TAP) block is a regional anesthetic that blocks sensory afferent nerve fibers that supply the anterior/lateral abdominal wall dermatomes of T6-L1. These sensory nerves travel below the internal oblique muscle in the plane above the transversus abdominis muscle. Traditionally, the technique is performed blindly by placing a needle through the triangle of Petit posteriorly until the needle reaches the TAP. Once the needle is in the appropriate plane, 20 mL of a long-acting local anesthetic, such as bupivacaine, is injected. More recent modifications include the use of ultrasound guidance to optimize precise placement and the use of diluted long-acting multivesicular liposomal bupivacaine (Exparel). Several studies have demonstrated the benefits of a TAP block during abdominal surgery. These benefits include decreased pain, opioid use, and nausea/vomiting, as well as faster return of bowel function. Complications include potential systemic toxicity due to dose of anesthetic delivered/inadvertent intravascular injection and intraperitoneal injection with possible injury to intraabdominal organs such as, the liver or spleen.

2017

How well did you know this?
1
Not at all
2
3
4
5
Perfectly
15
Q

A 42-year-old woman presents for fat grafting from the abdomen to the upper breast poles under intravenous sedation. Patient history includes implant-based breast reconstruction. She receives 1 g of intravenous cefazolin in the preoperative holding area. Ten minutes later, she has onset of generalized hives, flushing, and swelling of the lips. She reports dyspnea and has audible wheezing. The most appropriate next step is administration of epinephrine by which of the following routes?

A) Intradermal injection
B) Intramuscular injection
C) Intravenous bolus
D) Intravenous infusion
E) Subcutaneous injection

A

The correct response is Option B.

This patient meets the diagnostic criteria for anaphylaxis, manifested by acute onset of generalized hives and swelling along with either respiratory compromise or hypotension. Intramuscular injection of epinephrine, 0.3 to 0.5 mg, preferably in the mid-outer thigh, is the initial treatment in adults. Other treatments include administration of supplemental oxygen and rapid normal saline infusion. Absorption of epinephrine after subcutaneous injection may be too slow to reverse anaphylaxis. Bolus injections of epinephrine should be avoided because of the risks of dosing errors and overdose. Epinephrine infusion can be considered for anaphylaxis that is refractory to intramuscular injections, but it is not a first-line choice. Subcutaneous or intradermal injection would result in delayed absorption.

2017

How well did you know this?
1
Not at all
2
3
4
5
Perfectly
16
Q

A 154-lb (70-kg), 42-year-old woman undergoes wide-awake flexor tendon repair during local anesthesia. The surgical area is infiltrated with a total of 20 mL of 1% lidocaine with epinephrine. To minimize bleeding, which of the following is the optimal timing between injection of the anesthetic and initiation of the incision?

A) 5 minutes
B) 10 minutes
C) 15 minutes
D) 25 minutes
E) 45 minutes

A

The correct response is Option D.

Wide-awake hand surgery has been successfully performed for a number of procedures including nerve decompression, excision of soft-tissue tumors, fracture repair, arthrodesis, tendon repair, and fasciectomy for Dupuytren disease. The wide-awake technique allows the surgeon to perform the procedure without general anesthetic or sedation and often without the use of a tourniquet to minimize tourniquet pain. In order to successfully use this technique, a bloodless field is paramount, and allowing the epinephrine to achieve maximal vasoconstrictive effect is essential.

A prospective, randomized, triple-blind study was performed comparing the subcutaneous hemoglobin level at various times in patients after undergoing local anesthetic infiltration of lidocaine with epinephrine in one arm and plain lidocaine in the contralateral arm as the control. This study demonstrated that the lowest cutaneous hemoglobin level in the epinephrine group was obtained 25.9 minutes after injection. This is considerably longer than the 7 to 10 minutes often cited in the literature. The authors recommend waiting 25 minutes after injection if optimal visualization is desired.

2017

How well did you know this?
1
Not at all
2
3
4
5
Perfectly
17
Q

A 65-year-old woman is brought to the recovery room after undergoing brow lift surgery. Postoperatively, acute ST-elevation myocardial infarction is noted. Aspirin is administered. Frequent ventricular premature beats and occasional short runs of nonsustained ventricular tachycardia are noted on cardiac monitoring. Current blood pressure is 115/75 mmHg, heart rate is 65 bpm, and respiratory rate is 12/min. Oxygen saturation is 100%. Which of the following is the most appropriate next step in management?

A) Intravenous administration of amiodarone 150 mg over 10 minutes
B) Intravenous administration of flecainide 2 mg/kg body weight
C) Intravenous administration of lidocaine bolus 1 mg/kg body weight followed by continuous infusion at 1 mg/min
D) Replenishment of serum potassium to > 4.0 mEq/L and magnesium to > 2.0 mg/dL
E) Withhold administration of metoprolol

A

The correct response is Option D.

Asymptomatic ventricular premature beats (VPBs) and nonsustained ventricular tachycardia (NSVT) are not uncommon following myocardial infarction (MI), with reported incidences as high as 93% and 7%, respectively. These arrhythmias are thought to arise from transient abnormalities of cardiac automaticity or triggered activity in the region of ischemia or infarction. Routine suppression in the absence of hemodynamic compromise does not improve overall mortality, and because treatment may be associated with numerous potential adverse effects, antiarrhythmic medications are not recommended in this setting. Maintaining serum potassium concentration above 4.0 mEq/L and serum magnesium concentration above 2 mg/dL to normal ranges per general ACC/AHA guidelines followed by expectant management is the most reasonable option in this scenario.

In stark contrast, ventricular fibrillation (VF) and other symptomatic sustained ventricular tachyarrhythmias are thought to be indicators, if not potentiators, of ongoing myocardial ischemia and sudden cardiac death following acute MI. Consequently, rapid identification and treatment of these arrhythmias using defibrillation with or without antiarrhythmic medications, according to ACLS protocols, can be lifesaving. When indicated, amiodarone would be the recommended antiarrhythmic in this setting because it has not been associated with increased mortality. Both flecainide and lidocaine have been associated with increased mortality in the post-MI period and are therefore not recommended in the scenario described.

Intravenous administration of beta-adrenergic blockers has demonstrated numerous positive effects post MI including a reduced risk of VF. In the absence of other contraindications such as reactive airway disease and hypoperfusion, its ongoing administration in order to maintain a resting heart rate below 70 bpm is generally supported by the ACC/AHA guidelines for management of patients with STEMI.

Hypokalemia during an acute MI is a risk factor for VF, while hypomagnesemia can interfere with correction of hypokalemia, which is why ACC/AHA guidelines generally recommend maintaining normal concentrations in patients with an acute MI.

2017

How well did you know this?
1
Not at all
2
3
4
5
Perfectly
18
Q

During surgical procedures that do not involve the airway, which of the following oxygen delivery methods is most likely to increase the risk of fire in the operating room?

A) Endobronchial tube
B) Endotracheal tube
C) Laryngeal mask
D) Nasal cannula
E) Tracheostomy tube with cuff

A

The correct response is Option D.

For surgical procedures not involving the airway, open oxygen sources such as masks or nasal cannulae are most likely to increase the risk of fire in the operating room, as they may allow for trapping or pooling of an oxidizer-enriched atmosphere (oxygen with or without nitrous oxide).

The incidence of operating room fires in the United States is estimated to be around 600 cases per year. Fire requires the presence of three components: fuel, an oxidizer, and an ignition source. Common fuels in the operating room include alcohol-containing prepping agents, drapes and bandages, gowns and other personal protection equipment, petroleum jelly, etc. Ignition sources include electrocautery, lasers, fiberoptic light sources, and defibrillators. The two most common oxidizing agents in the operating room are oxygen and nitrous oxide.

Most protocols for assessment of risk of fire in the operating room take into account the presence of an open oxygen source, surgical site or incision above the xiphoid, and an available ignition source (such as electrocautery, laser, fiberoptic light source, etc).

For surgical procedures not involving the airway, closed-system oxygen sources using endotracheal/endobronchial tubes, tracheostomy cuffed tubes, or laryngeal masks convey less risk of fire, when compared with open oxygen sources.

2017

How well did you know this?
1
Not at all
2
3
4
5
Perfectly
19
Q

A 51-year-old woman undergoes bilateral breast reconstruction with free deep inferior epigastric artery perforator (DIEP) flaps. She is concerned about postoperative pain control. To achieve anesthetic effect, an infusion of 10 mL of 0.25% bupivacaine is administered bilaterally into the transversus abdominis plane (TAP) block space. Which of the following are the most appropriate anatomical landmarks to use when performing this technique?

A) Latissimus dorsi, external oblique, and iliac crest
B) Linea semilunaris, inferior epigastric vessels, and inguinal ligament
C) Quadratus lumborum, internal oblique, and 12th rib
D) Teres major, teres minor, and long head of the triceps

A

The correct response is Option A.

The transverse abdominis plane (TAP) block is a peripheral nerve block designed to anesthetize the nerves supplying the anterior abdominal wall (T6 to L1). Local anesthetic is injected between the internal oblique and the transverse abdominis muscles in the plane through which the sensory nerves pass. Use of this technique provides a suitable block to the multiple nerves supplying the abdominal wall with a single needle stick. The technique was first described in 2001 by Rafi using the anatomical landmarks of the inferior lumbar triangle (triangle of Petit): the anterior border of the latissimus dorsi muscle posteriorly, the posterior edge of the external oblique muscle anteriorly, and the superior edge of the iliac crest inferiorly.

The effectiveness of this technique has been demonstrated in the general surgery and OB-GYN literature with recent studies also being performed specifically in the harvest of abdominal free flaps for breast reconstruction. In a recent meta-analysis of this technique, the TAP block decreased narcotic use, provided more effective pain relief, and decreased opioid-related side effects including sedation and postoperative nausea and vomiting.

The teres major, teres minor, and long head of the triceps muscles form the borders of the triangular space through which the circumflex scapular artery can be found when harvesting the scapular flap.

Hesselbach triangle is bounded by the lateral border of the rectus muscle (linea semilunaris), the inferior epigastric vessels, and the inguinal ligament and is the space through which direct inguinal hernias pass.

The quadratus lumborum, internal oblique, and 12th rib are the anatomical landmarks of the superior lumbar triangle (Grynfeltt triangle) and a site of lumbar herniation.

2017

How well did you know this?
1
Not at all
2
3
4
5
Perfectly
20
Q

A 65-year-old man is evaluated for an increased serum potassium concentration of 6 mEq/L. ECG shows peaked T waves, decreased P wave amplitude, and prolonged QRS wave duration. Which of the following is the most appropriate initial step in management?

A) Administration of acetazolamide 250 mg intravenously
B) Administration of metoprolol succinate 20 mg in 4 mL nebulization, inhaled over 10 minutes
C) Administration of spironolactone 100 mg intravenously over 2 to 3 minutes, repeated after 5 minutes if ECG changes persist or recur
D) Administration of succinylcholine 30 g orally, and repeated every 2 hours as indicated
E) Administration of 10 units of regular insulin followed by 50 mL of 50% dextrose intravenously

A

The correct response is Option E.

Acute hyperkalemia is an electrolyte derangement that can result from a variety of scenarios including supratherapeutic potassium replacement, rhabdomyolysis, hemolysis, tumor lysis syndrome, severe sepsis, acute renal failure, and Addison disease. Among the most serious manifestations are progressive neuromuscular paralysis and progressive volatility in cardiac conduction terminating in cardiac arrest. The spectrum of cardiac collapse is evident on ECG starting with peaked T waves, decreased P wave amplitude, and prolonged QRS wave duration, followed by progressive blending of QRS and T waves into a sinusoidal ventricular fibrillation, followed by asystole.

Because cardiac arrest can occur at any point during ECG progression, hyperkalemia with ECG changes constitutes a medical emergency. Rapidly acting treatments aimed at sequestering or reducing serum potassium include administration of calcium, insulin with glucose, beta-2-adrenergeic agonists (e.g., albuterol, sodium bicarbonate), potassium wasting diuretics (e.g. furosemide), cation exchange resins (e.g., Kayexalate), and dialysis.

Of the options listed, only insulin with glucose is an accepted rapidly acting treatment for hyperkalemia. It works primarily by driving via the activity of the Na-K-ATPase pump in skeletal muscle. The effect begins in 10 to 20 minutes, peaks at 30 to 60 minutes, and lasts for four to six hours. Insulin can be given alone if serum glucose is > 250 mg/dL, and treatment can be repeated as needed. In all cases, serum glucose should be monitored to avoid acute hypoglycemia.

2017

How well did you know this?
1
Not at all
2
3
4
5
Perfectly
21
Q

An otherwise healthy 28-year-old woman undergoes emergency splenectomy because of isolated splenic rupture after sustaining blunt trauma. Examination shows hemodynamic instability. Postoperatively, the patient continues to experience hypotensive episodes that are difficult to manage. Total urine output is 200 mL over 12 hours. Tertiary survey identifies no ongoing hemorrhage or other injuries. Laboratory studies show:

Which of the following most closely represents the fractional excretion of sodium (FENa) in this patient?

A) 0.3%
B) 1.2%
C) 2.3%
D) 3.1%
E) 4.1%

A

The correct response is Option D.

Acute kidney injury (AKI, formerly called acute renal failure) is characterized by an abrupt decrease in renal function. The differential diagnosis can be categorized into prerenal, intrinsic renal, and postrenal causes. Of these, decreased renal perfusion (prerenal disease) and acute tubular necrosis (ATN) represent the two most common causes, accounting for 65 to 75% of all cases. The fractional excretion of sodium (FENa) has been shown in a variety of studies to more clearly differentiate between prerenal disease and ATN than other laboratory tests and guide patient management when taken in the context of history and physical examination. FENa measures the percent of filtered sodium and is calculated as:

FENa = [(UNa * PCr) / (PNa * UCr)]* 100%
where UNa and PNa represent urine and plasma concentrations of sodium, and UCr and PCr represent and urine and plasma concentrations of creatinine, respectively.

In general, a FENa below 1% suggests prerenal disease, where there is appropriate reabsorption of the majority of all filtered sodium in response to decreased renal perfusion. FENa above 2% reflects inappropriate salt wasting indicative of ATN. Of note, important limitations of the predictive value of a high FENa include diuretic therapy and chronic renal failure, by altering normal salt handling.

In the example given above, FENa = [(35 * 4.5) / (145 * 35)] * 100% = 3.1%

2017

How well did you know this?
1
Not at all
2
3
4
5
Perfectly
22
Q

A 28-year-old woman is scheduled to undergo office-based outpatient augmentation mammaplasty during general anesthesia. Medical history includes postoperative nausea and vomiting, and motion sickness. In addition to ondansetron 4 mg intravenously, preinduction administration of which of the following drugs is likely to be most effective in this patient?

A) Aprepitant 40 mg orally
B) Diphenhydramine 25 mg intravenously
C) Droperidol 1.25 mg intravenously
D) Metoclopramide 25 mg intravenously
E) Promethazine 25 mg orally

A

The correct response is Option A.

Aprepitant 40 mg should be administered orally 1 to 3 hours before induction of anesthesia.

Postoperative nausea and vomiting is a common complication after receiving general anesthesia. The overall incidence of this complication for all operations and patient populations is approximately 30% and increases to nearly 40% in patients treated at outpatient surgery centers. In patients who are at increased risk, the incidence of postoperative nausea and vomiting can be as high as 70 to 80%. Approximately 65% of all surgical procedures are now done on an outpatient basis and, if untreated, one third of these patients will have this complication. No antiemetic is universally effective in the prevention and treatment of postoperative nausea and vomiting. Each additional prophylactic antiemetic will decrease its incidence by approximately 25%.

Aprepitant is a highly selective, brain-penetrant, neurokinin-1 receptor antagonist with a long half-life and clinical efficacy against opioid-induced vomiting. It is the first in its class to be approved for this indication. Neurokinin-1 receptor antagonists act by blocking the binding of substance P at the neurokinin-1 receptor in the brain stem emetic center and gastrointestinal tract. Because of its mechanism of action and long half-life, it is most effective when administered as prophylaxis before surgery in patients who are at high risk for postoperative nausea and vomiting.

The remaining drugs all have short durations of action and are most effective when administered postoperatively. Ondansetron is a serotonin (5-HT3) receptor antagonist that exerts its effect on the chemoreceptor trigger zone and vagal afferents in the gastrointestinal tract. Droperidol blocks dopamine receptors and has an efficacy equivalent to that of ondansetron. Promethazine is a phenothiazine that acts primarily via central antidopaminergic mechanism. Metoclopramide blocks dopamine receptors in the central vomiting center and can block serotonin receptors at high doses.

2017

How well did you know this?
1
Not at all
2
3
4
5
Perfectly
23
Q

A 42-lb (19-kg), 5-year-old girl is brought to the emergency department because of multiple lacerations of the face after being bitten by a dog. Intravenous sedation is administered by the emergency room physician. The plastic surgeon initiates repair of the lacerations by infiltrating an infusion of 60 mL of 0.5% lidocaine with 1:200,000 epinephrine. Ten minutes after the procedure begins, the patient begins to have a seizure. Pulse oximetry and vital signs are within normal range. Which of the following is the most appropriate next step in management?

A) Administer epinephrine
B) Administer propofol
C) Initiate lipid emulsion therapy
D) Secure the airway and ventilate with oxygen
E) Transfer the patient to the PICU

A

The correct response is Option D.

The seizure in this scenario is related to lidocaine toxicity. A seizure in this scenario can be related to head trauma, but the mechanism of injury is not likely to have caused an intracranial injury. Hypoxia can also cause a seizure but is unlikely with a functioning pulse oximeter showing good oxygen saturation and normal vital signs. Lidocaine toxicity is the most likely cause. The safe limit of lidocaine dosage is 7 mg/kg when given with epinephrine. This patient received 60 mL of 0.5% lidocaine, or 300 mg (0.5% solution contains 5 mg/cc). The safe limit for this patient is 133 mg (19 x 7). The first signs of lidocaine toxicity in an awake and conscious patient is tinnitus. Seizures and cardiac arrhythmias follow. Treatment is supportive with establishment of an airway and vascular access. Lipid emulsion is provided intravenously due to the lipophilic nature of the local anesthetic and therefore hastens elimination. Epinephrine toxicity causes tachyarrhythmias and hypertension. Lidocaine is an amide anesthetic.

Administration of epinephrine would be appropriate if anaphylaxis from the preservative in the local anesthesia were suspected. This would present with hypotension and tachycardia. Propofol would stop the seizures, but is contraindicated due to its confounding effects on cardiac arrhythmias in the face of lidocaine toxicity. Lipid emulsion therapy will hasten the clearance of the excess of lidocaine, however, it is first most important to secure the airway and ventilate.

2017

How well did you know this?
1
Not at all
2
3
4
5
Perfectly
24
Q

A 50-year-old woman is scheduled to undergo bilateral deep inferior epigastric artery perforator (DIEP) flap breast reconstruction. A donor site regional block with liposomal bupivacaine is planned for analgesia. Which of the following injection locations is most likely to improve pain control and decrease narcotic use in this patient?

A) Between external oblique and internal oblique fascias
B) Between internal oblique and transversus abdominis fascias
C) Deep to rectus abdominis fascia
D) Deep to transversus abdominis fascia
E) Subcutaneous plane

A

The correct response is Option B.

The patient is undergoing a transversus abdominis plane (TAP) block. TAP blocks have become popular adjunctive pain control measures for a variety of abdominal surgeries. They can either be performed transcutaneously with ultrasound guidance or under direct visualization in the operating room prior to closure of the abdominal donor site in deep inferior epigastric artery perforator (DIEP) or transverse rectus abdominis musculocutaneous (TRAM) flap surgeries. The segmental innervation to be blocked in the TAP block is located between the internal oblique and transversus abdominis muscles. Either infiltration of liposomal bupivacaine or placement of epidural catheters in the TAP space has been described.

2016

How well did you know this?
1
Not at all
2
3
4
5
Perfectly
25
Q

A healthy 154-lb (70-kg), 30-year-old woman is undergoing liposuction of multiple sites. Five minutes after the local anesthetic infusion, she sustains a cardiac arrest. Local anesthetic systemic toxicity is diagnosed. After CPR is initiated, administration of which of the following is the most appropriate next step in management?

A) Adenosine
B) Intralipid
C) Propranolol
D) Vasopressin
E) Verapamil

A

The correct response is Option B.

This patient has sustained a cardiovascular collapse secondary to local anesthetic systemic toxicity (LAST). The maximum safe dose of lidocaine with epinephrine in this patient is 35 to 55 mg/kg.

Lipid Rescue protocol recommends an initial intralipid 20% bolus of 1.5 mL/kg over 1 minute. This should be followed immediately with a continuous infusion at 0.25 mL/kg/min. A single bolus is typical, but should be repeated or the infusion increased if spontaneous circulation fails to return or blood pressure declines.

Cardiovascular collapse from LAST differs from that secondary to myocardial ischemia. In LAST, raising the peripheral vascular resistance with vasopressors like vasopressin, can impair cardiac output and impede resuscitation. Epinephrine should be given in small doses (<1 mcg/kg). Pharmacologic agents that reduce contractility (beta blockers {propranolol}, calcium channel blockers {verapamil}, or propofol) should be avoided when there is evidence of cardiovascular instability.

Adenosine is not useful in the management of LAST patients. It is typically used to treat supraventricular tachycardia.

2016

How well did you know this?
1
Not at all
2
3
4
5
Perfectly
26
Q

A 55-year-old woman is evaluated for an incisional hernia that developed after a complicated right hemicolectomy 5 years ago. Medical history includes hypertension. BMI is 34 kg/m2. She reports shortness of breath when trying to scrub floors at work but is able to climb a flight of stairs or walk up a hill without symptoms. There are no other respiratory or cardiovascular issues. Cardiovascular examination shows no abnormalities. Recent electrocardiography, complete blood count, and basic metabolic panel show no abnormalities. Hernia repair with separation of components is being considered. Which of the following is the most appropriate next step in the preoperative cardiac assessment of this patient?

A) Cardiac catheterization
B) Echocardiographic exercise stress testing
C) Echocardiography at rest
D) Nuclear cardiac pharmacologic stress testing
E) No further cardiac testing is needed

A

The correct response is Option E.

No further preoperative cardiac testing is needed for this patient being considered for an elective, greater-than-low-risk procedure, without any evidence of active cardiac conditions or clinical risk factors, and with moderate functional capacity (metabolic equivalents, or METs, >= 4).

Assessment of left ventricular function at rest, whether by echocardiogram, radionuclide angiogram, or contrast ventriculography, has not been shown to be a consistent predictor of perioperative ischemic events. Cardiac stress testing, including by exercise or drug induced, is not indicated in this patient.

In 2007, the American College of Cardiology and the American Heart Association published updated guidelines on perioperative cardiovascular evaluation and care for noncardiac surgery (NCS), suggesting a stepwise algorithmic approach to perioperative cardiac risk assessment:

Is there need for emergency NCS? If yes, proceed with surgery.

Are there active cardiac conditions (unstable coronary syndromes, decompensated heart failure, significant arrhythmias, severe valvular disease)? If yes, evaluate and treat per specific guidelines before considering NCS.

Is the NCS considered low risk (e.g., superficial and ophthalmologic procedures)? If yes, proceed with low-risk NCS.

Does the patient have good functional capacity (METs >= 4) without symptoms? If yes, proceed with NCS.

Is there need for emergency NCS? If yes, proceed with surgery.

Three or more clinical risk factors undergoing vascular surgery: consider testing if it will change management

One or two clinical risk factors undergoing vascular surgery or one or more clinical risk factors undergoing intermediate-risk surgery: either proceed with surgery with heart rate control (beta blockade) or consider noninvasive testing if it will change management

Preoperative cardiac assessment should include an estimation of the patient’s functional capacity, based on his/her ability to perform a spectrum of common daily tasks. This has been shown to correlate well with maximum oxygen uptake by treadmill testing. Four METs is a common threshold used in many decision-making points in the perioperative cardiac evaluation.

2016

How well did you know this?
1
Not at all
2
3
4
5
Perfectly
27
Q

A 65-year-old woman is scheduled to undergo unilateral breast reconstruction with tissue expander. Medical history includes placement of a drug-eluting stent 4 months ago. Current medications include aspirin and clopidogrel. The surgical oncologist does not want to wait longer to perform the mastectomy. Which of the following is the most appropriate management of preoperative medications in this patient?

A) Continue both aspirin and clopidogrel
B) Discontinue aspirin and continue clopidogrel
C) Discontinue both aspirin and clopidogrel
D) Discontinue both aspirin and clopidogrel; start prophylactic enoxaparin
E) Discontinue clopidogrel and continue aspirin

A

The correct response is Option A.

A patient who underwent placement of a drug-eluting stent 4 months ago should continue on aspirin and clopidogrel for at least 6 months, except in cases with high risk of bleeding (intracranial or spine surgery). The combination of aspirin and clopidogrel appears to reduce the rates of cardiovascular ischemic events. Patients with coronary stents have an 8 to 10% risk of developing major adverse cardiovascular events and stent thrombosis after elective noncardiac surgery, which exceeds the 1 to 5% risk of major adverse cardiovascular events in non-stented patients having noncardiac surgery.

2016

How well did you know this?
1
Not at all
2
3
4
5
Perfectly
28
Q

A critically ill 69-year-old man is observed to have new-onset atrial fibrillation complicated by rapid ventricular response. Angina and hemodynamic instability with hypotension and acute ST depression is noted. Which of the following is the next step in management?

A) Emergency cardiac catherization with AV nodal ablation followed by permanent ventricular pacing
B) Heparin anticoagulation and emergency transesophageal echocardiography (TEE) to rule out left atrial thrombus
C) Immediate external electrical cardioversion
D) Intravenous administration of amiodarone with loading dose and titrated infusion until sinus rhythm is restored
E) Intravenous administration of a beta blocker or nondihydropyridine calcium channel blocker to slow the ventricular heart rate

A

The correct response is Option C.

Immediate cardioversion is recommended for patients with atrial fibrillation (AF) or atrial flutter with rapid ventricular response associated with hemodynamic instability such as hypotension, ongoing myocardial ischemia, or decompensated heart failure. Both chemical and electrical cardioversion can be attempted according to established protocols until sinus rhythm is restored with a goal of optimizing hemodynamic parameters. If initial cardioversion is unsuccessful, repeated attempts at cardioversion may be made after adjusting the location of the electrodes, applying pressure over the electrodes, or following administration of an antiarrhythmic medication according to Advanced Cardiac Life Support (ACLS) algorithms.

A trial of medical management for rate control is appropriate in the absence of hemodynamic instability. In general, beta blockers are the most common agents used for rate control, followed by nondihydropyridine calcium channel blockers such as diltiazem, as well as other agents such as digoxin and amiodarone. Patient comorbidities influence the choice of medical therapy to minimize risk of adverse events such as heart failure decompensation, exacerbation of chronic obstructive pulmonary disease, or acceleration of conduction in patients with preexcitation physiology.

Appropriate anticoagulation management around the time of elective cardioversion is essential for reducing thromboembolic risk. When AF has clearly existed at < 48 hours’ duration, it is common practice to perform cardioversion without transesophageal echocardiography (TEE) or antecedent anticoagulation. When AF has existed at >48 hours’ duration or duration is unknown, then anticoagulation for ? 3 weeks before and continuing for ? 4 weeks after cardioversion is recommended unless otherwise contraindicated. In case of relative contraindications to anticoagulation, TEE can be performed to help guide relative risk assessment related to thromboembolism of preexisting cardiac thrombi arising in the setting of prolonged atrial dysfunction. Importantly, when emergent cardioversion is indicated because of hemodynamic instability, initiation of anticoagulation and TEE should not delay acute interventions to stabilize the patient’s condition.

AV nodal ablation with permanent ventricular pacing can be considered to improve rate control when other attempts at maintaining rate control through medical management are unsuccessful. This intervention is usually reserved for elderly patients, because it leads to pacemaker dependency. However, patients with symptoms refractory to medical therapy who are treated with AV nodal ablation and permanent pacemaker implantation have demonstrated an improvement in cardiac symptoms, quality of life, and health care utilization.

2016

How well did you know this?
1
Not at all
2
3
4
5
Perfectly
29
Q

A 42-year-old woman is scheduled to undergo reduction mammaplasty. Medical history includes macromastia and von Willebrand disease. Preoperatively, administration of desmopressin (DDAVP) is planned for bleeding prophylaxis. The total preoperative dose is best determined by measuring the activity levels of vWF:RCoF (von Willebrand factor ristocetin cofactor) and which of the following coagulation factors?

A) II
B) V
C) VII
D) VIII
E) X

A

The correct response is Option D.

In surgical patients with von Willebrand disease (vWD) receiving desmopressin (DDAVP) for bleeding prophylaxis, total preoperative dose should be determined by preoperative measurement of activity levels of von Willebrand factor ristocetin cofactor (vWF:RCoF) and coagulation factor VIII.

The other listed coagulation factors are not associated with isolated vWD.

vWD is the most common congenital bleeding disorder, with estimated prevalence as high as 1.3%. It is caused by deficiency (types I and III) or dysfunction (type II) of von Willebrand factor (vWF), leading to impaired platelet adhesion and possibly lower levels of coagulation factor VIII. In its normal state, vWF is a plasma protein that mediates the initial adhesion of platelets at sites of vascular injury and also binds and stabilizes coagulation factor VIII in the circulation. Both vascular endothelial cells and platelets synthesize and store vWF.

DDAVP is a synthetic derivative of ADH (vasopressin, antidiuretic hormone) that is used for treatment and prophylaxis of bleeding in patients with vWD. It promotes release of stored vWF from endothelial cells into the plasma, being particularly effective in patients with partial quantitative deficiency of vWF, or type I (about 75% of cases of vWD). Patients who do not appropriately respond to DDAVP administration should receive vWF concentrate.

For bleeding prophylaxis for minor surgery, the preoperative goal should be to achieve vFW:RCoF and factor VIII activity levels of at least 30 IU/dL (preferably >50 IU/dL). Such levels should be maintained for 1 to 5 days postoperatively. For prophylaxis for major surgery, preoperative activity levels should be at least 100 IU/dL and maintained above 50 IU/dL for at least 7 to 10 days.

To decrease risk of perioperative thromboembolism, vWF:RCoF levels should not exceed 200 IU/dL and factor VIII activity should not exceed 250 IU/dL. Fluid restriction is advised for patients receiving DDAVP perioperatively, to avoid hyponatremia and seizures.

2016

How well did you know this?
1
Not at all
2
3
4
5
Perfectly
30
Q

A 55-year-old woman is evaluated for mild cellulitis after undergoing cosmetic excision of a facial mole 7 days ago. Cephalexin is prescribed. Approximately 30 minutes after taking her first dose, she returns to the office because of sudden and progressive onset of generalized hives, periorbital edema, and flushing. The patient appears confused and then collapses. Blood pressure is 85/50 mmHg. An audible wheeze is noted. Which of the following is the most appropriate next step in management?

A) Administration of albuterol nebulizer
B) Administration of diphenhydramine and ranitidine intravenously
C) Injection of epinephrine (1 mg/mL) 0.3 to 0.5 mg intramuscularly
D) Injection of methylprednisolone 125 mg intramuscularly
E) Rapid infusion of 0.9% saline 1 to 2 L intravenously

A

The correct response is Option C.

This patient is presenting with anaphylaxis. The first and most crucial step in managing acute anaphylaxis is the administration of epinephrine.

Anaphylaxis is a serious and life-threatening hypersensitivity reaction. Medications and insect stings are the most common triggers in adults, with beta-lactam antibiotic exposure among the most implicated medications. In this setting, anaphylaxis is diagnosed when two or more of the following are observed minutes to hours after exposure:

Skin-mucosal tissue reactions (such as generalized urticaria, swollen lip, pruritus, and flushing)

Respiratory compromise (such as dyspnea, wheeze, bronchospasm, stridor, and hypoxemia)

Hypotension or associated findings (such as syncope, confusion, collapse, and incontinence)

Persistent gastrointestinal reactions (such as crampy abdominal pain and vomiting)

Intramuscular epinephrine can be repeated every 5 to 15 minutes as needed. If given promptly, most patients respond to one or two doses. Simultaneous with this treatment, an emergency team can be summoned. The patient should be placed in a recumbent position with legs elevated and supplemental oxygen administered.

Volume resuscitation can be initiated to compensate for severe loss of intravascular volume that can accompany fluid shifts of untreated anaphylaxis and to support blood pressure not responsive to epinephrine.

Bronchodilators, H1 and H2 antihistamines such as diphenhydramine and ranitidine, and glucocorticoids such as methylprednisolone are all reasonable but second-line treatments for acute anaphylaxis.

Intubation should be performed immediately in the setting of progressive upper airway closure such as stridor and tongue edema, or if respiratory arrest is present. However, first-dose epinephrine administration should not be delayed for intubation. Epinephrine works rapidly to reduce airway edema that might otherwise prevent successful intubation, if not completely reverse the underlying etiology of cardiopulmonary collapse in anaphylaxis.

While cardiovascular disease is a risk factor for poor outcome from anaphylaxis, it is not a contraindication for epinephrine administration. It is generally accepted that the risk of death or brain damage from prolonged or undertreated anaphylaxis outweighs the risk of appropriately administered epinephrine. If there is inadequate response to initial intramuscular epinephrine and volume resuscitation, then intravenous epinephrine can be introduced in an intensive care setting by slow infusion at 2 to 10 mcg per minute and titrated to effect.

2016

How well did you know this?
1
Not at all
2
3
4
5
Perfectly
31
Q

A critically ill 65-year-old woman is brought to the intensive care unit, where she sustains respiratory arrest. Temperature is 101.3ºF (38.5ºC), heart rate is 105 bpm, and blood pressure is 85/60 mmHg. Hematocrit is 35%. She is immediately intubated. Noninvasive pulse oximetry is initiated to monitor oxygen saturation (SaO2). Which of the following conditions is most likely to alter pulse oximetry values in this patient?

A) Anemia
B) Fever
C) Hypotension
D) Tachycardia
E) Tachypnea

A

The correct response is Option C.

Hypotension is most likely to alter pulse oximetry values by reducing peripheral arterial blood flow.

Pulse oximetry measures the relative transmission of light at two wavelengths that differ significantly when passed through loaded versus non-loaded hemoglobin (e.g., oxyhemoglobin versus deoxyhemoglobin). In order to provide values that correlate with arterial oxygen saturation levels as opposed to tissue bed or venous saturation levels, standard pulse oximeters require pulsatile blood to distinguish transmission at the peak of arterial pulsation relative to baseline transmission levels. Thus, pulse oximetry measurements will change both with changes in hemoglobin oxygen saturation and with conditions that interfere with the device’s ability to detect pulsatile blood flow.

Other conditions that alter pulse oximetry measurements by reducing the detection of fluctuations from arterial blood flow include peripheral vasoconstriction from hypothermia and vasopressor and interference from motion, such as tremors or shivering. Incorrect sensor application, highly calloused skin, and nail polish can also affect measurements by interfering with transmission readings.

Because standard pulse oximetry only measures the relative difference in transmission between oxygenated and deoxygenated hemoglobin and not the absolute value of oxygenated hemoglobin, anemia does not significantly affect pulse oximetry values within physiologic ranges.

Tachycardia, tachypnea, and fever do not directly affect pulse oximetry values.

2016

How well did you know this?
1
Not at all
2
3
4
5
Perfectly
32
Q

A 63-year-old man with end-stage renal failure is evaluated for unilateral leg swelling and shortness of breath 2 days after undergoing bilateral axillary hidradenitis excision. Current medication includes aspirin. Oxygen saturation on room air is 88%. During diagnostic evaluation, empiric administration of which of the following drugs is most appropriate?

A) Intravenous heparin
B) Oral clopidogrel
C) Oral rivaroxaban
D) Oral warfarin
E) Subcutaneous enoxaparin

A

The correct response is Option A.

The patient has a presumed diagnosis of deep vein thrombosis (DVT) and pulmonary embolism (PE). Enoxaparin and rivaroxaban are contraindicated in patients with renal failure. Warfarin is used for long-term treatment of established DVT. Clopidogrel is an antiplatelet inhibitor that inhibits blood clots in coronary artery disease, peripheral vascular disease, cerebrovascular disease, and prevents myocardial infarction. Heparin is safe in renal failure patients and is indicated for treatment in acute DVT/PE.

2016

How well did you know this?
1
Not at all
2
3
4
5
Perfectly
33
Q

A 69-year-old man is evaluated in the intensive care unit 5 days after undergoing abdominal wall reconstruction for a multiple-recurrence ventral hernia. Medical history shows no cardiac disease. Temperature is 103°F (39°C), heart rate is 110 bpm, and mean arterial pressure (MAP) is 50 mmHg. Airway and ventilation are secured, and supplemental oxygen is initiated. Hemoglobin concentration is 9 g/dL. Which of the following is the most appropriate next step in management?

A) Initiation of inotropic support
B) Initiation of vasopressor support
C) Introduction of a pulmonary artery catheter to monitor wedge pressures
D) Volume resuscitation with crystalloid
E) Volume resuscitation with packed red blood cells

A

The correct response is Option D.

In this patient with septic shock, once airway and breathing are secured, the next most appropriate step in management is to restore effective circulation and perfusion to peripheral tissues starting with a trial of volume resuscitation with crystalloid.

Signs of impaired end organ perfusion in septic shock include hypotension (e.g., mean arterial pressure <70 mmHg), tachycardia (e.g., heart rate >100 bpm), warm flushed skin giving way to cool clammy skin as blood flow is redirected to core organs, obtundation, and an elevated serum lactate concentration (e.g., >1 mmol/L).

First-line therapy for restoration of tissue perfusion is volume resuscitation using intravenous crystalloid targeted to physiologic end points while monitoring for clinical or radiographic evidence of either cardiogenic or non-cardiogenic pulmonary edema (ie, ARDS).

The addition of pulmonary artery catheters has not been shown to improve outcomes in the routine management of septic shock and is associated with increased complications.

As in the given scenario, evidence and expert opinion do not support the transfusion of blood products greater than a hemoglobin concentration of 7 g/dL in the absence of concurrent hemorrhagic shock, cardiac history, or active myocardial ischemia. For example, a recent multicenter randomized study involving 998 patients with septic shock reported no significant difference in mortality or rate of ischemic events between patients transfused when hemoglobin concentration was <7 g/dL compared with patients transfused when hemoglobin was <9 g/dL. Their former (more restrictive) transfusion trigger resulted in 50% fewer red blood cell transfusions compared with the more liberal strategy.

Vasopressors (e.g., norepinephrine) are second-line agents in the treatment of septic shock refractory to trial of volume resuscitation as long as intravenous fluids successfully improve perfusion without impairing gas exchange.

Inotropic agents (e.g., dobutamine) are also second-line agents to initial volume resuscitation in this scenario. They may be useful with refractory shock in the setting of diminished cardiac output.

2016

How well did you know this?
1
Not at all
2
3
4
5
Perfectly
34
Q

A 5-year-old boy is brought to the emergency department 45 minutes after accidentally injecting his palm with epinephrine from an auto-injector (EpiPen). On physical examination, the ring finger is soft and pale, and capillary refill time is poor. A small puncture mark is noted on the flexor surface of the palm just proximal to the metacarpophalangeal (MCP) joint. Which of the following is the most appropriate next step in management?

A) Application of a hot pack
B) Emergent operative exploration
C) Subcutaneous injection of nifedipine
D) Topical nitroglycerin paste
E) Observation

A

The correct response is Option E.

Accidental self-injection of epinephrine with an EpiPen occurs in 1 in 50,000 syringes. There have been no documented cases of digital necrosis following injection, and observation is indicated in this case. The effect of epinephrine’s vasoconstriction lasts for approximately 90 minutes and will likely resolve on its own.

Phentolamine has been described as a reversal agent for epinephrine and has shown clinical efficacy, but there has been no study to show that there are better outcomes with phentolamine injection compared with observation. Also, the added volume of injection with phentolamine could cause pressure necrosis and the timing between EpiPen injection and definitive treatment would usually be outside of the 90 minutes it would take for the epinephrine to wear off on its own.

Topical nitroglycerin paste and subcutaneous injection of calcium channel blockers such as nifedipine have not shown to be viable treatment modalities. Application of a hot pack can lead to increased tissue damage and burns and is not indicated. Emergent exploration is indicated for compartment syndrome and is not indicated in this case where the finger is soft.

2015

How well did you know this?
1
Not at all
2
3
4
5
Perfectly
35
Q

A 35-year-old man undergoes a 90-minute rhytidectomy procedure with intravenous sedation. The patient smokes 10 cigarettes daily, but is otherwise healthy. Which of the following factors most likely places this patient at increased risk for postoperative nausea and vomiting?

A) Duration of procedure
B) Gender
C) History of cigarette smoking
D) History of postoperative nausea and vomiting
E) Type of anesthesia

A

The correct response is Option D.

Risk factors for postoperative nausea and vomiting include: female gender, nonsmoking status, prior history of postoperative nausea/vomiting/motion sickness, use of volatile anesthetics/general anesthesia, opioid/narcotic use, facial rejuvenation procedures, and long duration of surgery.

Several measures can be taken to decrease postoperative nausea and vomiting. A thorough history with identification of risk factors can aid stratification of patients preoperatively. Use of long-acting local anesthetic agents, nonsteroidal anti-inflammatory drugs (NSAIDs), and cyclooxygenase-2 selective inhibitors can decrease the need for postoperative opioid use. Avoidance of nitrous oxide, especially in combination with fentanyl and volatile inhalational gases, and multimodal use of serotonin antagonists combined with other antiemetic agents can also decrease postoperative nausea and vomiting.

2015

How well did you know this?
1
Not at all
2
3
4
5
Perfectly
36
Q

A 37-year-old man, who is American Society of Anesthesiologists (ASA) Class 2, comes to the office for evaluation and treatment of human immunodeficiency virus (HIV)–associated lipodystrophy. The patient has a CD4 count of 100 cells/mm3. Autologous fat grafting is planned. Which of the following factors is most likely to increase this patient’s risk of postoperative complications?

A) ASA Class
B) CD4 cell count
C) HIV seropositivity
D) Percutaneous surgery

A

The correct response is Option B.

Higher American Society of Anesthesiologists (ASA) class has been identified as a risk factor for postoperative complications in HIV-positive patients in multiple studies. A patient who is ASA Class 2 has only mild systemic disease. Increasing class number indicates increasing severity of disease (Class 3 – severe systemic disease, Class 4 – severe systemic disease that is a constant threat to life).

Acquired immunodeficiency syndrome (AIDS) is diagnosed when the CD4 count is <200 cells/mm3 or with acquisition of an AIDS-defining illness. An absolute CD4 count of <200 cells/mm3 has been associated with increased risk of complications including wound infections.

HIV seropositivity alone has been found not to be an independent risk factor for postoperative complications.

Percutaneous surgery, such as fat grafting, has not been associated with increased risk of infection in HIV patients, nor has skin incisional surgery. Transoral mucosal incisional surgery has been found to be associated with a significantly greater risk of wound infection in HIV patients.

A viral load greater than 10,000 copies/mL suggests that antiretroviral therapy is no longer effective and has been identified as an independent risk factor for complications.

2015

How well did you know this?
1
Not at all
2
3
4
5
Perfectly
37
Q

A 35-year-old woman comes to the office for consultation on augmentation mammaplasty. During preoperative workup, she reports that her mother has a history of malignant hyperthermia. The patient has never undergone surgery. Which of the following anesthetic agents is most appropriate for this surgery?

A) Desflurane
B) Halothane
C) Isoflurane
D) Propofol
E) Succinylcholine

A

The correct response is Option D.

Propofol can be safely used in patients with a suspected diagnosis of malignant hyperthermia.

Malignant hyperthermia is a rare, life-threatening inherited skeletal muscle disorder that shows symptoms of hypermetabolic reaction to volatile anesthetic gases and depolarizing muscle relaxants. The incidence is between 1 in 5000 to 1 in 100,000 anesthetic encounters. Mortality rates have decreased from 70% to less than 5% as awareness of this condition has led to accurate diagnosis and treatment.

Malignant hyperthermia is genetically transmitted through an autosomal dominant inheritance pattern with variable penetrance. In obtaining a medical history, it is important to document family history of adverse outcomes to general anesthesia. If it is reported that a first-degree relative has had a malignant hyperthermia crisis or susceptibility, then the patient should not be exposed to triggering agents. Anesthetic agents that trigger malignant hyperthermia include: halothane, enflurane, isoflurane, desflurane, sevoflurane, and succinylcholine. Nitric oxide can be used if the anesthesia machine is “vapor-free” and contains no traces of volatile gas. Other safe agents include nondepolarizing muscle relaxants (such as vecuronium, rocuronium, and pancuronium), all ester and amide local anesthetics, ketamine, propofol, etomidate, barbiturates, opiates, and benzodiazepines.

Although it is safe to undergo minor procedures with administration of a topical or local anesthetic agent, patients undergoing complex procedures with intravenous sedation, general anesthesia, or major conduction blockade should be referred to an accredited ambulatory surgical center or hospital. If symptoms are recognized in the operating room (high temperature, increased end-tidal CO2, muscle rigidity), rapid treatment with dantrolene sodium is the highest priority. Acute episodes may require stopping the procedure and transfer to an intensive care unit.

2015

How well did you know this?
1
Not at all
2
3
4
5
Perfectly
38
Q

A 42-year-old woman, with a history of anaphylactic reaction to procaine, comes to the office for consultation regarding augmentation mammaplasty. Anesthetics that contain which of the following should be avoided in this patient?

A) Acetaldehyde
B) Epinephrine
C) Iodine
D) Methylparaben
E) Para-aminobenzoic acid

A

The correct response is Option E.

True allergic reactions to local anesthetics are rare. Type I hypersensitivity reactions may include anaphylaxis, and are modulated by immunoglobulin E. Both ester-based local anesthetics, such as procaine, and amide-based local anesthetics may induce an allergic response. The most likely allergen is para-aminobenzoic acid (PABA). This compound consists of a benzene ring substituted with an amino group at the 4-position (leading to the analogous label of 4-aminobenzoic acid) as well as a carboxyl group. Preservatives such as methylparaben have also been shown to cause allergic reactions to local anesthetics, but are less likely than allergic reactions caused by PABA.

Epinephrine can induce vasoconstriction and increase the safe dosage of administered local anesthetic, though it would not be expected to cause an allergic response itself. Iodine and acetaldehyde should not be found in local anesthetic mixtures.

2015

How well did you know this?
1
Not at all
2
3
4
5
Perfectly
39
Q

An otherwise healthy 52-year-old woman with a family history of cardiac disease undergoes suction-assisted lipectomy of the flanks, thighs, and abdomen using a tumescent technique. She returns to the emergency department 6 hours after discharge because of slurred speech and restlessness. Which of the following is the most likely diagnosis?

A) Fat embolism
B) Lidocaine toxicity
C) Parietal stroke
D) Pulmonary embolism
E) Third spacing

A

The correct response is Option B.

Because lidocaine absorbs slowly from fat, infiltrate solutions that contain up to 35 mg/kg of lidocaine are generally considered safe. Nonetheless, lidocaine toxicity is still a risk of the procedure. In tumescent solution with epinephrine, peak plasma lidocaine levels occur approximately 10 to 14 hours after infiltration, and thus, the presentation 6 hours after discharge is consistent with peak plasma concentration.

Lidocaine toxicity has symptoms of neurologic or cardiac toxicity. In the early stages, the complications are primarily neurologic and can include slurred speech, restlessness, tinnitus, and a metallic taste, as well as numbness of the mouth. As the concentrations increase, the neurologic concentrations become more severe, and can progress to muscle twitching, seizures, and cardiac arrest. Treatment of lidocaine toxicity is supportive.

Fat embolism presents as a petechial rash, respiratory dysfunction, and cerebral dysfunction, and the symptoms usually appear 24 to 48 hours after surgery. Pulmonary embolism presents as leg pain and edema, tachycardia, and low-grade fevers.

Parietal strokes usually cause sensory symptoms, self-perception anomalies, and left-right agnosia. Third spacing refers to fluid shifts into interstitial spaces and can cause edema, hypotension, and decreased cardiac output.

2015

How well did you know this?
1
Not at all
2
3
4
5
Perfectly
40
Q

A 40-year-old woman with a history of severe postoperative nausea and vomiting is scheduled for exchange of bilateral breast tissue expanders for permanent silicone implants. Use of which of the following medications is most likely to decrease the chance of postoperative nausea?

A) Bupivacaine
B) Fentanyl
C) Isoflurane
D) Midazolam
E) Nitrous oxide

A

The correct response is Option A.

Addition of local anesthetics during general anesthesia, whether by subcutaneous, tumescent, or regional block infiltration, can result in decreased dosage requirements of the common sedatives and analgesics that can result in nausea and emesis.

Common anesthetic agents that promote nausea and emesis include opioids (fentanyl, hydromorphone, morphine) and inhalationals (halothane, isoflurane, nitrous oxide). Propofol is currently the most commonly used intravenous agent. It does not appear to directly result in nausea, but it has limited analgesic effects. Therefore, effective anesthesia with propofol requires addition of opioid narcotics (which cause nausea) and/or local anesthetics such as lidocaine and bupivacaine (which may decrease the narcotic requirement).

Midazolam is a sedative-hypnotic that has anxiolytic and amnesic effects, both of which are helpful adjuncts to the surgical patient experience. Nausea is possible with midazolam, but less commonly reported than with narcotic and inhalational agents.

The cause of postoperative nausea and vomiting is multifactorial and not fully understood. Strategies for prevention include:

Recognition of high-risk patients (females, nonsmokers, history of motion sickness, previous postoperative nausea, general anesthesia)
Pre- and postoperative treatment with multiple modalities
(such as scopolamine, ondansetron, aprepitant, corticosteroids) Supplemental intraoperative oxygen
and hydration

2014

How well did you know this?
1
Not at all
2
3
4
5
Perfectly
41
Q

A 54-year-old woman has onset of ventricular fibrillation and severe hypotension 5 minutes after 30 mL bupivacaine 0.5% is administered to the ankle for postoperative pain control during reconstruction of the foot. After initiation of cardiopulmonary resuscitation, intravenous administration of which of the following is the most appropriate management?

A) Atropine
B) Dantrolene
C) Flumazenil
D) Lipid emulsion
E) Metoprolol

A

The correct response is Option D.

The most appropriate management of acute bupivacaine toxicity is a bolus and infusion of 20% lipid emulsion. Every facility where local anesthetic is used in large doses should have a lipid rescue kit clearly labeled and available should the need arise. Although lipid rescue mechanism of action is not completely understood, it may be that the added lipid in the bloodstream acts as a “sink,” allowing for the removal of lipophilic toxins from affected tissues. Major local anesthetic toxicity can have such symptoms as sudden loss of consciousness, tonic-clonic seizures, hypertension followed by progressive hypotension, tachycardia, ventricular fibrillation, bradycardia, asystole, and cardiac arrest. Arrhythmias may be refractory to treatment, and resuscitation may be prolonged, sometimes requiring more than 1 hour.

In the event of a local anesthetic toxicity event, airway management, seizure suppression, and, if needed, cardiopulmonary resuscitation should be performed. Alert the nearest facility having cardiopulmonary bypass capability and administer 20% lipid emulsion (values in parentheses are for 70 kg) as follows:

Bolus 1.5 mL/kg intravenously over 1 minute (~100 mL)
Continuous infusion 0.25 mL/kg/min (~500 mL over 30 minutes)
Repeat bolus every 5 minutes for persistent cardiovascular collapse
Double infusion rate if blood pressure returns but remains decreased
Continue infusion for a minimum of 30 minutes

Although beta-adrenergic blockers may be useful in treating the excitatory cardiovascular phase of local anesthetic toxicity, the potential to progress to more advanced phases with myocardial depression and collapse preclude their routine use. In addition to lipid emulsion, the treatment for local anesthetic–induced cardiac toxicity is generally supportive, and may include amrinone, closed-chest cardiac massage, and cardiopulmonary bypass.

Flumazenil is used to reverse the effects of benzodiazepine toxicity.

Dantrolene is administered in the acute treatment of malignant hyperthermia.

Atropine and dopamine are administered as part of the Advanced Cardiac Life Support protocol for bradycardia or asystole and would not be used in the scenario described.

2014

How well did you know this?
1
Not at all
2
3
4
5
Perfectly
42
Q

A 16-year-old boy who has asthma is brought to the emergency department 3 hours after accidentally injecting the index finger of the nondominant hand with his epinephrine auto-injector. On examination, the finger is cool, pale, and painful. Which of the following drugs works to competitively antagonize the sympathomimetic effects of epinephrine?

A) Lidocaine
B) Marcaine
C) Nitroglycerin paste
D) Phentolamine
E) Prostacyclin

A

The correct response is Option D.

Epinephrine use in hand surgery is becoming more common as is the inadvertent self-injection by people who carry epinephrine injectors (EpiPens). Typically, there is little treatment needed other than supportive care. However, when concern for tissue viability is raised or there is marked pain, subcutaneous phentolamine is the drug of choice to reverse the sympathomimetic effects of epinephrine. Plain lidocaine (typically 2% or more) will cause vasodilation but by a different mechanism than the reversal of the epinephrine. Topical nitroglycerin paste has been used for reversal of vasospasm, but again, a different mechanism is used.

Marcaine is an amide anesthetic that inhibits sodium ion channels. It is not an antagonist of epinephrine.

2014

How well did you know this?
1
Not at all
2
3
4
5
Perfectly
43
Q

A 53-year-old woman comes to the office for removal of multiple nevi. On injection of lidocaine, which of the following signs and symptoms is most likely to suggest lidocaine toxicity in this patient?

A) Bronchospasm
B) Hypertension
C) Tachycardia
D) Tinnitus
E) Urticaria

A

The correct response is Option D.

Signs and symptoms of lidocaine toxicity include dizziness, agitation, lethargy, tinnitus, metallic taste, perioral paresthesia, slurred speech, euphoria, hypotension, and bradycardia.

Tachycardia is not a sign of lidocaine toxicity. Bradycardia is more common.

Bronchospasm and urticaria are not signs of lidocaine toxicity.

2014

How well did you know this?
1
Not at all
2
3
4
5
Perfectly
44
Q

A 37-year-old woman is scheduled to undergo skin-sparing mastectomy of the left breast with immediate deep inferior epigastric perforator (DIEP) flap reconstruction. Which of the following interventions is most likely to decrease this patient’s postoperative narcotic needs?

A) Application of a preoperative compression garment
B) Hypnosis
C) Ondansetron therapy
D) Oxycodone therapy
E) Pregabalin therapy

A

The correct response is Option E.

Pregabalin (Lyrica) has been shown to decrease narcotic needs after breast surgery. It is a gamma-aminobutyric acid (GABA) analogue and decreases GABA in the brain and acts primarily as an anticonvulsant. It is more potent than gabapentin as an analgesic.

Compression garments would be useful for post-mastectomy lymphedema but would not decrease narcotic needs. Hypnosis might decrease narcotic use slightly, but the effects are negligible. Oxycodone is a narcotic which does not decrease narcotic needs. The use of ondansetron (Zofran) has not been shown to reduce post-operative narcotic requirements.

2019

How well did you know this?
1
Not at all
2
3
4
5
Perfectly
45
Q

After an uneventful breast reconstruction with an abdominal flap, a healthy 45-year-old woman is started on ketorolac as part of her multi-modality pain control regimen. Inhibition of which of the following is the primary mechanism of action for the increased bleeding time associated with this drug?

A) Lipoxygenase
B) Nitric oxide
C) Prostacyclin
D) Prostaglandins
E) Thromboxane A2

A

The correct response is Option E.

The primary mechanism of action for the increased bleeding time associated with the use of ketorolac is the inhibition of thromboxane A2. Ketorolac is a non-steroidal anti-inflammatory drug (NSAID). Most NSAIDs inhibit the activity of cyclooxygenase-1 (COX-1) and cyclooxygenase-2 (COX-2), and thereby the synthesis of thromboxane A2. This produces a systemic bleeding tendency by impairing thromboxane-dependent platelet aggregation and consequently prolonging the bleeding time. It is thought that inhibition of COX-2 leads to the anti-inflammatory, analgesic, and antipyretic effects through the inhibition of formation of prostaglandins and prostacyclin. Inhibition of lipoxygenase and nitric oxide are not the primary mechanisms associated with NSAID-related platelet inhibition.

2019

How well did you know this?
1
Not at all
2
3
4
5
Perfectly
46
Q

A 10-year-old boy undergoes surgical repair of microtia of the right ear. During cartilage rib harvest, the right thorax is damaged with visualization of the lung. After repair of the pleura, Valsalva maneuver is performed with no evidence of an air leak. An intraoperative chest x-ray is negative for pneumothorax. Several minutes later in the PACU, the patient becomes hypotensive and tachypneic, and his oxygen saturation decreases to the mid-80s, despite use of a non-rebreather mask. Which of the following is the most appropriate next step in management?

A) Draw arterial blood gas
B) Intubation
C) Needle decompression of the right chest
D) Open the chest incision
E) Portable chest x-ray study

A

The correct response is Option C.

The patient shows all the signs of tension pneumothorax, and although the precise etiology is unclear, the patient requires decompression. Intubation will not help relieve the tension and pressure, with decreased venous return jeopardizing hemodynamic stability.

Immediate chest x-ray is inappropriate because of the time required.

Opening the chest incision is not a good option because it requires surgical equipment, general anesthesia, and cannot be completed in a timely fashion.

Needle decompression at the second intercostal is the standard of care to decompress a tension pneumothorax. After oxygen saturation and hemodynamics are stabilized, definitive treatment of pneumothorax can be pursued. This would include placement of chest tube to low suction and serial chest x-ray to monitor the progress of the lung inflation.

Arterial blood gas will not help make the diagnosis and potentially will delay the appropriate intervention.

2019

How well did you know this?
1
Not at all
2
3
4
5
Perfectly
47
Q

A 36-year-old health-care worker sustains a needle-stick injury from a hepatitis C–seropositive patient. Immediate testing for anti-HCV antibodies and confirmatory immunoassays for HCV-RNA are performed. Initial follow-up testing after exposure should be performed at which of the following time periods?

A) 1 week
B) 3 weeks
C) 6 weeks
D) 12 weeks
E) 24 weeks

A

The correct response is Option C.

It is recommended that follow-up retesting be done at 6 weeks, 3 months, and 6 months in known HCV exposure cases. Tests at 1 or 3 weeks would possibly lead to false negative results. There is no advantage in waiting beyond 6 weeks.

2019

How well did you know this?
1
Not at all
2
3
4
5
Perfectly
48
Q

A 70-year-old man is in the recovery room after undergoing radial forearm free flap reconstruction for squamous cell carcinoma of the tongue. He has a 30-pack year history of smoking. The patient is ventilated with a tracheostomy tube. Two days postoperatively, sedation is turned off for an hour with the goal of weaning the patient off mechanical ventilation. He becomes agitated and delirious. Which of the following drugs should be avoided in this patient during the postoperative period?

A) Albuterol
B) Diphenhydramine
C) Neostigmine
D) Propranolol
E) Varenicline

A

The correct response is Option B.

Postoperative delirium is an acute brain dysfunction that is characterized by changes in levels of consciousness, inattention, and disorganized thinking. There are two types of delirium. Delirium can manifest with hyperactive signs (agitation, restlessness), or hypoactive signs (lethargy, inattentiveness). It is very common in hospitalized patients, with 60 to 80% of mechanically ventilated patients and 20 to 50% of patients with a lower severity of illness developing delirium at some point during their hospitalization.

For patients at risk of postoperative delirium, benzodiazepines and antihistamines should be avoided, as these medications could exacerbate the symptoms.

2019

How well did you know this?
1
Not at all
2
3
4
5
Perfectly
49
Q

A 75-year-old woman with type 1 diabetes mellitus undergoes closure of a sternotomy wound using pectoralis major muscle flaps. On postoperative day 2, her plasma creatinine level has increased to 2.2 from 1.1 mg/dL preoperatively. The patient is hemodynamically stable in the ICU, and her central venous pressure is within normal range. An intravenous infusion of normal saline is initiated. Which of the following is the most appropriate next step in management?

A) Administration of a diuretic

B) Discontinuation of enteral nutrition and initiation of parenteral nutrition

C) Discontinuation of protein intake

D) Infusion of low-dose (<2.5 ?g/kg/min) dopamine intravenously

E) Plasma glucose control protocol

A

The correct response is Option E.

This patient has acute kidney injury (AKI) after a surgical procedure. International practice guidelines recommend insulin therapy for targeted glucose control in critically ill patients. Although the Kidney Disease – Improving Global Outcomes (KDIGO) task force recommended a plasma glucose target of 110 to 149 mg/dL, the latest recommendation by the Surviving Sepsis Campaign is for an upper blood glucose level not higher than 180 mg/dL.

Other recommendations for prevention and treatment of AKI by the 2012 KDIGO Clinical Practice Guideline included: Isotonic crystalloids rather than colloids (albumin or starches) as initial management for expansion of intravascular volume in patients at risk for or with AKI; Avoding restriction of protein intake with the aim of preventing or delaying initiation of renal replacement therapy (RRT); Administration of 0.8 to 1.0 g/kg/d of protein in non-catabolic AKI patients without need for dialysis; 1.0 to 1.5 g/kg/d in patients with AKI on RRT; and up to a maximum of 1.7 g/kg/d in patients on continuous renal replacement therapy (CRRT) and in hypercatabolic patients; Providing nutrition preferentially via the enteral route in patients with AKI; Not using diuretics to prevent AKI; Not using diuretics to treat AKI, except in the management of volume overload; Not using low-dose dopamine to prevent or treat AKI

2019

How well did you know this?
1
Not at all
2
3
4
5
Perfectly
50
Q

A 67-year-old man undergoes ventral hernia repair and abdominal wall reconstruction with component separation. On postoperative day 5, the patient develops a cough; temperature is 39.0°C (102.2°F). Chest x-ray study shows right middle lobe pneumonia. Antibiotic therapy is promptly initiated. Despite adequate fluid resuscitation, the patient becomes hypotensive (mean arterial pressure < 65 mmHg). Which of the following blood tests is most appropriate to establish the suspected diagnosis of septic shock?

A) Albumin
B) C-reactive protein
C) Lactate
D) Plasminogen
E) White blood cell count

A

The correct response is Option C.

Obtaining a serum lactate level is the most appropriate next step for the diagnosis of septic shock in this scenario. Patients with septic shock can be clinically identified by having both of two criteria:

Vasopressor requirement to maintain a mean arterial pressure of 65 mmHg or greater and

Serum lactate level greater than 2 mmol/L (>18 mg/dL) in the absence of hypovolemia.

In 1991, a consensus task force developed initial definitions that focused on the prevailing view at the time that sepsis resulted from a host’s systemic inflammatory response syndrome (SIRS) to infection. SIRS was defined by the presence of two or more of four criteria, including body temperature, heart rate, respiratory rate, and white blood cell count. Despite their known limitations, these definitions remained mainly unchanged for almost three decades. In 2016, the Society of Critical Care Medicine and the European Society of Intensive Care Medicine sponsored a task force to review the definition of sepsis and its management guidelines (Sepsis-3).

Sepsis is now defined as a life-threatening organ dysfunction caused by a dysregulated host response to infection. This organ dysfunction can be represented by an increase in the Sequential [Sepsis-related] Organ Failure Assessment (SOFA) score of two points or more. Another measure called quick SOFA (qSOFA), although less robust, may be more practical for providers diagnosing sepsis in the non-ICU setting. qSOFA incorporates altered mentation (GCS <15), systolic blood pressure of 100 mmHg or less, and respiratory rate of 22/min or greater.

Septic shock is a subset of sepsis with profound circulatory and cellular/metabolic dysfunction, associated with a higher risk of hospital mortality than with sepsis alone (40% versus 10%, respectively).

The term “severe sepsis,” previously defined as sepsis complicated by organ dysfunction, has been incorporated into the current definition of sepsis and abandoned.

2019

How well did you know this?
1
Not at all
2
3
4
5
Perfectly
51
Q

Which of the following patients is eligible to be an organ donor?

A) 30-year-old HIV-positive patient
B) 10-year-old whose family does not want to donate
C) 42-year-old with a diagnosis of Creutzfeldt-Jakob disease
D) 49-year-old with a recent diagnosis of stage IV melanoma

A

The correct response is Option A.

Minors are neither eligible for nor able to receive organ donation without the consent of a parent or guardian.

Creutzfeldt-Jakob disease or any other prion disease is a contraindication for organ donation of any kind.

Metastatic cancer is a contraindication for organ donation.

HIV status is no longer a contraindication to donate or receive an organ, provided the donor and recipient are both HIV positive.

2019

52
Q

An 81-year-old man with peripheral vascular disease undergoes coverage of exposed vascular prosthesis in the groin with a rectus femoris muscle flap. On postoperative day 2, the patient has sudden onset of chest tightness and becomes unresponsive, with no palpable pulse. CPR is promptly initiated. The defibrillator monitor shows ventricular fibrillation. Which of the following is the most appropriate next step after shock delivery (electrical defibrillation)?

A) Capnometry
B) Chest compressions for 2 minutes
C) Endotracheal intubation
D) Intravenous administration of adenosine
E) Pulse/rhythm check

A

The correct response is Option B.

According to current Advanced Cardiac Life Support (ACLS) guidelines, CPR should be resumed immediately after shock delivery, without pausing for a rhythm or pulse check. It should begin with chest compressions and continue for 2 minutes, after which the rhythm should be checked and the cycle (shock/CPR 2 min/check) repeated if there is no return of spontaneous circulation (ROSC).

Increasing emphasis has been placed on the importance of continuous “high-quality” chest compression (5 cm sternal depression, 100 to 120/min), to maximize tissue perfusion and probability of ROSC. A 30:2 compression:ventilation rate is recommended in the absence of an endotracheal or supraglottic airway. Otherwise, 10 breaths per minute should be delivered with continuous chest compressions.

There are no studies directly addressing the timing of advanced airway placement and outcome during resuscitation from cardiac arrest. Although insertion of an endotracheal tube during ongoing chest compressions is possible, in most instances intubation is associated with interruption of compressions for many seconds. Particularly, patients with witnessed cardiac arrest from ventricular fibrillation or pulseless ventricular tachycardia may benefit from a few uninterrupted cycles of CPR prior to placement of an advanced airway.

Adenosine is not indicated in the treatment of adult cardiac arrest.

Capnometry/capnography requires placement of an endotracheal tube.

2019

53
Q

A 45-year-old woman undergoes abdominoplasty in an ambulatory surgery center. After induction of general anesthesia, the patient’s end tidal carbon dioxide level increases, her heart rate increases to 160 bpm, and her arms become rigid. Which of the following are the physiologic abnormalities associated with this condition?

A) Hyperkalemia, hyperphosphatemia, metabolic acidosis
B) Hyperkalemia, hyperphosphatemia, metabolic alkalosis
C) Hyperkalemia, hypophosphatemia, metabolic acidosis
D) Hypokalemia, hyperphosphatemia, metabolic acidosis
E) Hypokalemia, hyperphosphatemia, metabolic alkalosis

A

The correct response is Option A.

Malignant hyperthermia is an inherited myopathy that is autosomal dominant with variable penetrance. Anesthetic agents that trigger malignant hyperthermia include halothane, enflurane, isoflurane, desflurane, sevoflurane, and succinylcholine. These agents trigger an earlier calcium release into the skeletal muscle, resulting in an abnormal buildup of calcium in the myoplasm. This flood of calcium causes the muscle to remain in a contracted state, producing high levels of lactic acid, carbon dioxide, phosphate, and heat. The resulting physiologic changes are metabolic acidosis, hypercapnia, hyperphosphatemia, and fever in a patient experiencing malignant hyperthermia. The treatment of malignant hyperthermia is discontinuation of volatile agents and succinylcholine, dantrolene, treatment of hyperkalemia and metabolic acidosis, and transfer to an acute care hospital.

2019

54
Q

Administration of prophylactic antibiotics is most appropriate for which of the following surgical procedures?

A) Abdominoplasty
B) Blepharoplasty
C) Brachioplasty
D) Mastopexy
E) Rhytidectomy

A

The correct response is Option D.

The ASPS recently published the first consensus statement/guidelines for antibiotic prophylaxis in plastic surgery which is based on comprehensive systematic review of the available evidence.

Systemic antibiotic prophylaxis is recommended for clean-contaminated, contaminated, or dirty plastic surgery of the head and neck, orthognathic/mandibular, septoplasty/rhinoplasty, hand and upper limb, and skin. Antibiotic prophylaxis is also recommended to reduce surgical-site infection for clean plastic surgery of the breast. Antibiotic prophylaxis is not recommended to reduce surgical-site infection in clean surgical cases of the head and neck, orthognathic/mandibular area, hand and upper limb, skin, and abdominoplasty.

With the exception of cosmetic breast surgery, clean operations have not been shown to benefit from routine antibiotic prophylaxis. Clean-contaminated and contaminated plastic surgical procedures do benefit from the use of antibiotic prophylaxis. The duration of antibiotic use should generally be limited to a single preoperative dose because studies have generally showed no benefit for longer term antibiotic prophylaxis.

As far as choosing the antibiotic, it should have activity against the most frequently encountered microorganisms in postoperative surgical-site infections. Cefazolin as a single dose preoperatively is the most commonly recommended agent and would be considered appropriate in most cases. In the event of allergy or intolerance, clindamycin or vancomycin may be appropriate alternatives.

2019

55
Q

A 65-year-old woman remains intubated in the intensive care unit after undergoing a prolonged operation. Arterial blood gas analysis shows respiratory acidosis. An increase in respiratory minute ventilation is planned. Minute ventilation is calculated by multiplying the respiratory rate and which of the following parameters?

A) Inspiratory capacity
B) Residual volume
C) Tidal volume
D) Total lung capacity
E) Vital capacity

A

The correct response is Option C.

Minute ventilation is calculated by multiplying respiratory rate and tidal volume. Tidal volume is the amount of air/gas displaced during each quiet breath, using no extra inspiratory (“deep breath”) or expiratory effort.

Minute ventilation is an important concept in mechanical ventilation because of its inverse relationship with blood carbon dioxide levels. The caveat of this relationship is that not all inhaled air/gas takes part in gas exchange, whether because it remains in the conductive airways (ventilator tubing, endotracheal tube, trachea, etc) or it reaches alveoli that are not adequately perfused. The volume of air/gas that does not take part in gas exchange is called dead space.

Residual volume is the volume of air still remaining in the lungs after the most forcible expiration possible.

Inspiratory capacity is the volume of air that enters the lungs during the most forcible inspiration possible, starting at rest. Inspiratory reserve volume equals inspiratory capacity minus tidal volume, or the difference between the deepest breath and a quiet breath.

Expiratory reserve volume is the amount of air that can still be expired after a quiet expiration ends. It requires contraction of expiratory chest wall muscles, as opposed to quiet expiration, which is passive.

Vital capacity is the total amount of air that can be forcefully expired from the lungs after the most forcible inspiration possible. It represents the addition of inspiratory reserve, tidal, and expiratory reserve volumes.

Total lung capacity is the combination of vital capacity and residual volume.

2019

56
Q

A 30-year-old woman undergoes augmentation mammaplasty in an office-based operating room. Intravenous midazolam and fentanyl are used, and a lidocaine field block is administered. An hour later, while in the recovery room, the patient experiences disorientation, muscle twitching, and light-headedness. Administration of which of the following drugs is the most appropriate next step in management?

A) Dantrolene
B) Fat emulsion
C) Flumazenil
D) Naloxone
E) Propofol

A

The correct response is Option B.

This patient is experiencing symptoms of lidocaine toxicity. Lidocaine toxicity occurs within a few minutes after injection but can occur up to 60 minutes after injection. The maximum dose of lidocaine without epinephrine is 4.5 mg/kg and with epinephrine is 7 mg/kg. Symptoms of lidocaine toxicity can range from central nervous system (CNS) excitement (circumoral/tongue numbness, metallic taste, light-headedness, dizziness, visual and auditory disturbances, disorientation, drowsiness), and at higher doses CNS depression (muscle twitching, convulsions, unconsciousness, coma, respiratory depression and arrest, cardiovascular depression and collapse). Cardiovascular manifestations include chest pain, shortness of breath, palpitations, hypotension, and syncope. Of the options presented, fat emulsion (Intralipid) is the treatment of choice. Flumazenil is the treatment for benzodiazepine overdose, naloxone is for opioid overdose, dantrolene is for malignant hyperthermia, and propofol is used for induction and maintenance of general anesthesia.

2019

57
Q

A 63-year-old woman has a superficial surgical site infection at her abdominal incision two weeks following deep inferior epigastric perforator (DIEP) breast reconstruction. The CDC categorizes a “superficial incisional surgical-site infection” as occurring within how many days of the procedure?

A) No more than 7 days
B) No more than 14 days
C) No more than 30 days
D) No more than 60 days
E) No more than 90 days

A

The correct response is Option C.

Superficial incisional surgical-site infection is defined by the following criteria:

Date of event for infection occurs within 30 days of a procedure
AND
involves only skin and subcutaneous tissue of the incision
AND
the patient has at least one of the following:

A. purulent drainage at incision site
B. positive cultures from the incisions or underlying tissue
C. superficial incision that is deliberately opened by a surgeon or other designee with the following signs or symptoms: pain or tenderness; localized swelling; erythema; or heat
D. diagnosis by the surgeon or attending physician designee

2019

58
Q

Which of the following factors has been shown to have the greatest impact on infection reduction in trauma patients with a lower-extremity open fracture?

A) Definitive irrigation and debridement, and initiation of negative pressure wound therapy within 12 hours of injury
B) Early intramedullary reaming and nail fixation
C) Evaluation of the patient at a level I trauma center
D) Initial debridement of the wound by a senior surgeon
E) Intravenous administration of an antibiotic within 3 hours of injury

A

The correct response is Option E.

Early administration of antibiotics (less than 3 hours after injury) has been shown to be the most important determinant of infection prevention after traumatic open fractures of the lower extremity; this is more influential than other factors including time to initial washout, seniority of surgeon involved, as well as severity of the extremity trauma. Guidelines differ by institution but at least a cephalosporin is advocated with consideration given to additional gram-negative plus or minus anaerobic coverage in grossly contaminated wounds.

In a large multi-institutional study of open fractures, Pollak et al, showed a significant decrease in infection rate with either early direct admission (<2 hours) or transfer (<11 hours) to a level I trauma center, though this was only true for the Gustilo Type III open tibial fracture subgroup and not all open fractures. The authors hypothesized that early transfer resulted in earlier administration of antibiotics, though this was not directly compared.

While prompt debridement is important in obtaining wound control in traumatic lower-extremity open fractures, no clear advantage has been shown to debridement within 6 hours versus 24 hours, with the accepted standard that this can typically wait until daylight hours in the setting of timely prophylactic antibiotic administration.

Intramedullary reaming has not been shown to decrease infection risk in open tibial fractures, although early skeletal reduction and stability will minimize ongoing soft tissue damage.

2019

59
Q

A 25-year-old woman comes to the office for excision of a skin lesion with local anesthesia. Medical history includes multiple food and drug allergies. Shortly after the procedure is initiated, the patient reports itchy eyes and runny nose and becomes very anxious. Within the next 5 minutes, the patient develops diffuse skin erythema and shortness of breath, which progresses to respiratory stridor. Intravenous access has not been established. Intramuscular administration of which of the following drugs is the most appropriate next step in management?

A) Diphenhydramine
B) Epinephrine
C) Ketamine
D) Midazolam
E) Triamcinolone

A

The correct response is Option B.

Epinephrine is the first line of treatment for patients with anaphylaxis and should be administered intramuscularly (anterolateral thigh) as soon as the diagnosis is made. In a setting where an exact dose can be drawn up, the recommended dosage is 0.01 mg/kg (maximum dose of 0.5 mg), administered intramuscularly every 5 to 15 minutes as necessary to control symptoms. It can also be administered more frequently than every 5 minutes if needed.

Greater emphasis has been placed on early administration of epinephrine in the management of anaphylaxis. The mean time to cardiac or respiratory arrest from medication-induced anaphylaxis in a hospital setting has been shown in one study to be only 5 minutes, with only 14% of patients receiving epinephrine prior to arrest.

Besides intramuscular epinephrine, recommended immediate interventions for anaphylaxis include an assessment of airway, breathing, and circulation; intravenous access; supplemental oxygen; monitoring; and placing the patient in supine position. Depending on the initial response, other interventions include establishing an airway, rapid intravenous fluid infusion, intravenous epinephrine infusion, bronchodilators, steroids, H1/H2 antihistamines, and glucagon.

Ketamine and midazolam (versed) are not indicated for the treatment of anaphylaxis. Diphenhydramine (Benadryl) and other antihistamine drugs are recommended as a second line of therapy for anaphylaxis. The same applies to systemic corticosteroids.

2019

60
Q

Which of the following must be present in order to initiate a brain death examination?

A) Absence of deep tendon reflexes
B) Anoxia
C) Cause of brain death
D) Hypothermia
E) Ventilatory dependence with muscle relaxation (neuromuscular blockade)

A

The correct response is Option C.

Brain death is a permanent and irreversible state. There will be no return of cerebral or cortical function.

Hypothermia is not required for brain death examination; it needs to be reversed for at least 4 hours (up to 24 hours) to establish brain death.

Neuromuscular blockade must be reversed and patient must have normal peripheral muscle response to peripheral nerve stimulus in order to undergo brain death examination. Brain death examination includes elicitation of brain reflexes, which can be muted by neuromuscular blockade.

Anoxic brain injury must be observed for at least 24 hours prior to beginning brain death examination.

Cause or reason for brain death must be established prior to beginning brain death examination.

Absence of deep tendon reflexes is not required to initiate the examination.

2019

61
Q

A 56-year-old man is scheduled to undergo excision of a lower extremity melanoma during regional anesthesia. Current medications include lisinopril and occasional motrin. He does not smoke cigarettes. Which of the following factors increases the risk of postoperative nausea and vomiting in this patient?

A) Age over 50 years
B) Male gender
C) Nonsmoking status
D) Use of anti-inflammatory medications
E) Use of local anesthetic

A

The correct response is Option C.

Risk factors for postoperative nausea and vomiting fall into four categories: patient-related, anesthesia-related, surgery-related, and other factors.

Patient-related predictors are: female sex, non-smoking status, history of postoperative nausea and vomiting/motion sickness, genetics, age of 50 years or younger, and obesity (BMI greater than 30 kg/m2).

Anesthesia-related predictors are: postoperative opioids, inhalational anesthetics, and nitrous oxide.

Surgery-related predictors are: surgery duration and surgery type.

Other factors including high patient anxiety and postoperative pain.

2019

62
Q

An otherwise healthy 35-year-old woman, gravida 3, para 3, presents for abdominoplasty. When combined with non-opioid analgesics and/or NSAIDs, which of the following is the most appropriate pain management for this patient?

A) Administration of epidural anesthetic
B) Infiltration of wound with liposomal bupivacaine
C) Intraoperative dexamethasone administration
D) Intraoperative ketamine infusion
E) Intraoperative lidocaine infusion

A

The correct response is Option B.

Enhanced recovery after surgery (ERAS) protocols are multimodal, multidisciplinary perioperative care pathways designed to achieve rapid recovery after surgery. These pathways include consensus recommendation for postoperative analgesia. In addition, improved postoperative pain control can be obtained with emphasis on the use of procedure-specific pain management. Some procedures have a higher propensity for persistent postoperative pain which generally are neuropathic in origin, ie: mastectomy, thoracotomy, hernia repair, abdominal wall surgeries. The primary goal of an optimal pain therapy is to provide “dynamic” pain relief (pain relief during movement) that would allow early ambulation while reducing opioid consumption. The ideal multimodal analgesic technique would include a local/regional analgesic (wound infiltration or peripheral nerve block) as the principal component because they provide excellent dynamic pain relief. Liposomal bupivacaine (Exparel) allows delivery of bupivacaine for 96 hours with a single local administration. There can be significant pain relief with the combination of wound infiltration with liposomal bupivacaine acetaminophen and NSAIDs or COX2 inhibitors as multimodal analgesic regimens. Epidural analgesia, dexamethasone, lidocaine and ketamine infusions all have demonstrated postoperative pain relief and reduction in opioid requirements to varying degrees. But local/regional analgesia (wound infiltration) should be used as the first-line analgesic therapy, which should be combined with acetaminophen, NSAIDs or COX1 inhibitors.

2020

63
Q

A 30-year-old woman with a history of Crohn disease undergoes ventral hernia repair. BMI is 31 kg/m2. Which of the following is the greatest risk factor for postoperative nausea and vomiting in this patient?

A) Elevated body mass index
B) Perioperative fasting
C) Placement of nasogastric tube
D) Supplemental oxygen
E) Young age

A

The correct response is Option E.

Postoperative nausea and vomiting remains a significant problem after cosmetic and reconstructive plastic surgery. Reported studies on the condition list incidences as high as 56%, whereas a meta-analysis found that the overall incidence was 28.3%.

For many plastic surgery procedures, general inhalational anesthesia and narcotic pain control are required and may predispose patients to postoperative nausea and vomiting (PONV).The effects can be disastrous including hematoma, incisional dehiscence, respiratory compromise, pain, longer hospital stay, slower recuperation, and patient dissatisfaction.

A number of risk factors have been associated with a positive overall incidence of PONV. Patient-specific risk factors for PONV in adults include female sex, a history of PONV, non-smoking status, young age, general versus regional anesthesia, postoperative opioids, and type of surgery.

Other factors have been disproven including placement of a nasogastric tube, peri-operative fasting, obesity, and supplemental oxygen.

2020

64
Q

A 32-year-old man undergoes unilateral hand transplantation. Tacrolimus for immunosupression is initiated. On routine evaluation 6 months postoperatively, a chronic increase in creatinine and a reduction in glomerular filtration rate is noted. Which of the following is the most appropriate next step in management?

A) Discontinuation of antihypertensive drugs
B) Discontinuation of dyslipidemia drugs
C) Initiation of oral corticosteroid therapy
D) Reduction of calcineurin inhibitor trough levels
E) Tissue biopsy of the transplanted hand

A

The correct response is Option D.

This patient is experiencing nephrotoxicity and chronic kidney disease (CKD) from tacrolimus. Calcineurin inhibitor nephrotoxicity is a well-known phenomenon posttransplantation, and close monitoring of kidney function is essential. There are multiple described pathways of kidney damage secondary to calcineurin inhibitors including irreversible damage to all compartments of the kidney (glomeruli, arterioles, and tubule-interstitium).

Krezdorn et al. evaluated 99 recipients of facial or extremity transplantation and concluded that kidney dysfunction represents a major complication posttransplantation in vascularized composite allografts and recommends pretransplant, peritransplant, and posttransplant strategies to reduce kidney damage. These include identifying patients at risk for CKD. Pretransplantation recommendations include treating existing renal conditions, avoiding hypotension and hypertension, limiting nephrotoxic drugs, limiting intravenous contrast, and avoiding hypovolemia. Peritransplantation recommendations include minimizing use of nephrotoxic agents, avoiding hypovolemia, and limiting ischemia time. Postoperative recommendations include minimizing tacrolimus exposure including reduction of trough levels, treating hypertension, treating hyperglycemia, treating dyslipidemia avoiding intravenous contrast, and limiting potentially nephrotoxic drugs such as nonsteroidal anti-inflammatory drugs.

Therefore, in this patient, tacrolimus troughs should be reduced. Initiation of oral corticosteroids is not appropriate. The patient should remain on antihypertensive and dyslipidemia drugs.

2020

65
Q

A 45-year-old woman with breast cancer is scheduled to undergo bilateral mastectomy. Immediate breast reconstruction with deep inferior epigastric perforator (DIEP) flaps is planned. Which of the following factors is most likely to increase this patient’s risk for microsurgical thrombotic complications?

A) BRCA-2 genetic mutation

B) Caprini Risk Assessment Model score of 5

C) History of prior irradiation following lumpectomy

D) History of prior thrombotic event

E) Sickle cell trait

A

The correct response is Option D.

Virchow recognized a triad of factors that predispose to intravascular thrombosis. These are stasis in blood flow, endothelial (intimal) damage, and intrinsic hypercoagulability. One recent review identifies a personal history of prior thrombotic event as perhaps the single greatest risk factor of a hypercoagulable state. Other known hypercoagulable disorders that can be identified by specific blood test include Factor V Leiden mutation, prothrombin gene (20210A) mutation, protein C deficiency, protein S deficiency, antithrombin III (AT3) deficiency, lupus anticoagulant, anticardiolipin antibodies, and anti-beta-2 glycoprotein 1 antibodies. These appear to increase the risk of microsurgical thrombotic complications as well. However, the severity of the increased risk for each remains to be fully elucidated. Another published series of 41 patients showed an 80% free flap success rate in patients with identified hypercoagulable states. Therefore, a thorough preoperative evaluation of patients for microvascular procedures may help to identify those at increased risk for thrombotic complications, guiding patient selection and perioperative anticoagulation therapy.

The Caprini Risk Assessment Model (RAM) is used to assess venous thromboembolism (VTE) risk, and its use has been validated in plastic and reconstructive surgery patients. The Caprini RAM score for the patient in this question is 5 (2 risk factor points for major surgery over 45 minutes, 2 points for presence of malignancy, and 1 point for ages 41 to 60). According to the recommendations of the American Society of Plastic Surgeons VTE Task Force Report, one should consider postoperative chemoprophylaxis for VTE for this patient who is at intermediate risk. However, data are lacking, which would support the use of the Caprini Risk Assessment Model as a tool to stratify risk of microvascular thrombotic complications.

This examination contains test materials that are owned and copyrighted by the American Society of Plastic Surgeons. Any reproduction of these materials or any part of them, through any means, including but not limited to, copying or printing electronic files, reconstruction through memorization or dictation, and/or dissemination of these materials or any part of them is strictly prohibited. Keep printed materials in a secure location when you are not reviewing them and discard them in a secure manner, such as shredding, when you have completed the examination.

BRCA-2 genetic mutation is a heritable condition that significantly increases lifetime risk for breast and ovarian cancer. However, it has not been shown to play any role in risk for thrombotic events.

Similarly, sickle cell trait (heterozygous carrier of the sickle cell mutation in the hemoglobin-beta gene) has not been shown to increase risk for microsurgical thrombotic complications.

While chest wall irradiation might negatively impact the recipient chest wall vessels for deep inferior epigastric perforator (DIEP) flaps, the impact of radiation following a lumpectomy is very unlikely to be as significant as that of a prior personal thrombotic event.

2020

66
Q

A 25-year-old man presents for consultation for surgical resection of a large abdominal pannus. The patient’s mother has a history of malignant hyperthermia. Which of the following best describes the inheritance pattern of this condition?

A) Autosomal dominant
B) Autosomal recessive
C) Mitochondrial
D) X-linked dominant
E) X-linked recessive

A

The correct response is Option A.

Malignant hyperthermia is an autosomal dominant trait, thus, based on Mendelian Genetics, if one parent has a confirmed diagnosis, their biological children will have a 50% chance of inheritance of the affected gene.

Malignant hyperthermia is a potentially life-threatening condition. Individuals with this inherited myopathy present with a hypermetabolic reaction to potent volatile anesthetic gases, such as halothane, enflurane, isoflurane, sevoflurane, and desflurane. Individuals are also susceptible to the depolarizing muscle relaxant succinylcholine. The preoperative workup for an individual with suspected malignant hyperthermia is the Caffeine-Halothane Contracture Test. In this test, a piece of skeletal muscle is excised, and its contractile properties are determined when exposed to the ryanodine receptor agonist halothane and/or caffeine. Abnormal contractile activity is indicative of susceptibility. Based on basic Mendelian Genetics, the risk of inheritance is 50%.

Given this pattern of inheritance, x-linked, autosomal recessive and mitochondrial inheritance are incorrect answers.

2020

67
Q

A 54-year-old woman undergoes abdominoplasty and hysterectomy. Postoperative rivaroxaban is initiated for prevention of deep venous thrombosis. Three days later, she presents for follow-up, and a hematoma requiring drainage is noted. This outcome is most likely related to the fact that rivaroxaban blocks the coagulation cascade in which of the following ways?

A) Binds factors II, VII, IX, and X
B) Prevents conversion of prothrombin to thrombin
C) Prevents degranulation of platelets
D) Prevents thrombin from activating fibrinogen
E) Prevents tissue factor:factor VIIa complex formation

A

The correct response is Option B.

The direct anticoagulation agent rivaroxaban (Xarelto) is a direct anticoagulant, which acts within the clotting cascade by blocking Factor Xa, which, along with Factor Va, helps convert prothrombin to thrombin. This step in the cascade is where the intrinsic and extrinsic pathways intersect and the common pathway that leads to fibrinogen being cleaved to fibrin and stabilized by factor XIIIa as a cross-linked clot.

Aspirin is a drug that interferes with platelet function. Coumadin affects the vitamin K dependent factors II (prothombin), VII, IX and X. Heparin prevents clot propagation by blocking thrombin-mediated activation of fibrinogen to fibrin. The primary benefits of the direct anticoagulation agents over coumadin include no need for monitoring and equivalence in efficacy across many clinical situations. It is excreted by the kidney, so dosages must be altered or the drug avoided in renal failure. A U.S. Food & Drug Administration–approved reversal agent, recombinant coagulation factor Xa (Andexxa), is now available on the market for life-threatening or uncontrolled bleeding in patients using direct anticoagulants affecting factor Xa, like rivaroxaban. Prior to this, administration of fresh frozen plasma (FFP) was the antidote of choice.

2020

68
Q

A 69-year-old man is scheduled to undergo excision and direct closure of a basal cell carcinoma of the chest. Medical history includes aspirin 81 mg daily for primary prevention of cardiovascular disease. Compared with patients not taking aspirin prophylaxis, this patient is most likely at risk for which of the following complications?

A) Hematoma
B) Worse cosmetic outcome
C) Wound dehiscence
D) Wound infection
E) None of the above

A

The correct response is Option E.

Systematic review drawing from 30 studies and more than 14,000 patients undergoing minor cutaneous surgery firmly supports continuation of aspirin therapy in all minor cutaneous surgery, as patients on aspirin monotherapy are at no greater risk of hemorrhagic complications than those on no agents. While a case-by-case risk profiling in all patients on aspirin therapy is prudent, the preponderance of evidence favors meticulous hemostasis over aspirin cessation in prevention of bleeding complications in minor cutaneous surgery.

There is no reported association between aspirin utilization and increased risk of wound dehiscence, wound infection, or cosmetic outcome.

2020

69
Q

A 45-year-old man presents with significant closed head injury. Maintenance fluids are initiated because of progressive nausea with reduced oral intake. CT scan of the head shows diffuse cerebral edema. Administration of which of the following solutions is most appropriate in this patient?

A) Dextrose 5% in water
B) Hypertonic (3%) saline
C) Hypotonic (0.45%) saline
D) Physiologic (0.9%) saline
E) Ringer’s lactate

A

The correct response is Option B.

The patient is showing signs of symptomatic progressive intracranial pressure following head trauma. Hypertonic infusion therapy can be used in this scenario to decrease intracranial pressure and curtail progressive cerebral edema. Hypertonic (3%) saline fits into conventional algorithms for treatment of symptomatic acute intracranial hypertension along with hyperventilation, mannitol, diuretics, and surgical decompression. Hypertonic solutions have been shown to decrease intracranial pressure with greater efficiency than mannitol in early stages of trauma for patients with evolving brain injury.

Ringer’s lactate and physiologic (0.9%) saline solution would not prevent or counteract progressive cerebral edema following head trauma. Hypotonic solutions (such as dextrose 5% in water and 0.45% saline solutions) are broadly contraindicated in patients who suffer severe traumatic brain injury, because they may lower serum osmolarity and exacerbate cerebral edema.

2020

70
Q

A 69-year-old woman presents to the emergency department because she has had increasing redness and pain in her left reconstructed breast for the past 24 hours. Blood pressure is 80/40 mmHg, heart rate is 130 bpm, and respiratory rate is 32/min. Initial plasma lactate level is 5.2 mmol/L. Ultrasonography shows no fluid collections within the breast. Intravenous fluid resuscitation is started. Administration of intravenous antibiotics is most appropriate within how many hours?

A) 1
B) 3
C) 6
D) 12
E) 24

A

The correct response is Option A.

The 2016 Surviving Sepsis Campaign guidelines strongly recommend that administration of intravenous antibiotics be initiated as soon as possible after recognition and within 1 hour for both sepsis and septic shock.

In the presence of sepsis or septic shock, increasing delays in administration of appropriate antibiotics are associated with increasing mortality and detrimental effects on secondary endpoints, such as length of hospital stay, acute kidney injury, acute lung injury, and the Sepsis-Related Organ Assessment score.

Although data suggest that optimal outcomes are achieved by the earliest possible administration of appropriate antibiotics following recognition of sepsis, 1 hour was recommended as a reasonable minimal target, considering multiple patient and organizational factors that may cause delay.

2020

71
Q

A 30-year-old woman, gravida 1, para 0, who is at 36 weeks’ gestation presents to the emergency department after sustaining blunt trauma injury in a motor vehicle collision. Physical examination shows a visible seat belt sign, but she is hemodynamically stable. Secondary assessment is unremarkable. Which of the following is the most appropriate next step?

A) Determine maternal Rh status
B) Discharge with obstetric follow-up
C) Initiate scalp monitoring of the fetus
D) Perform diagnostic peritoneal lavage
E) Perform MRI of the abdomen/pelvis

A

The correct response is Option A.

Traumatic placental injury can cause maternal-fetal hemorrhage in 10 to 30% of pregnant trauma patients. The majority of these are subclinical, without measurable effects to the fetus. However, as little as 0.001 milliliter of Rh-positive fetal blood can cause alloimmunization of an Rh-negative mother. Therefore, determination of Rh status is mandatory for all pregnant trauma patients and all Rh-negative of these should be given anti-D immune globulin (IgG). A single dose administered within 72 hours from injury may provide protection against sensitization in up to 90% of the cases. Higher doses may be necessary if it is determined that transplacental hemorrhage was in excess of 30 milliliters.

Simply discharging the patient for a future obstetric follow up is not acceptable. At least 4 hours of electronic fetal monitoring should be provided to all pregnant trauma patients with >= 23 weeks gestation.

Internal electronic fetal heart rate monitoring (with an electrode in the scalp) is not indicated at this time. Instead, external monitoring could be done using a dedicated Doppler ultrasound device.

MRI of the abdomen/pelvis and diagnostic peritoneal lavage are not indicated in the scenario described. Radiographic imaging generally indicated for trauma evaluation, including abdominal computed tomography, should not be deferred or delayed due to concerns of fetal exposure to radiation. Ultrasonography (FAST) should be part of the secondary survey in all pregnant trauma patients.

2020

72
Q

A 65-year-old woman undergoes ventral hernia repair with component separation and mesh placement. On postoperative day two, the patient suddenly becomes unresponsive and has no palpable pulse. Cardiopulmonary resuscitation (CPR) is promptly started and a cardiac monitor is attached. Cardiac tracing is consistent with pulseless electrical activity (PEA). Administration of which of the following drugs is most appropriate in this patient?

A) Adenosine
B) Atropine
C) Diltiazem
D) Dopamine
E) Epinephrine

A

The correct response is Option E.

Epinephrine is the initial drug of choice in the acute management of this patient with pulseless electrical activity (PEA).

Desirable effects of epinephrine in the treatment of cardiac arrest include vasoconstriction (alpha-1 adrenergic) and increase in cardiac output (beta-1 adrenergic). According to current Advanced Cardiac Life Support guidelines, epinephrine should be administered to a patient in cardiac arrest as soon as the electrocardiographic diagnosis of either PEA or asystole is made. The intravenous dose is 1 mg every 3 to 5 minutes, always followed by a 20 mL normal saline flush. Cardiopulmonary resuscitation should not be halted for drug administration. The drug can also be administered via intraosseous access or through the endotracheal tube.

The other drugs listed are not indicated in the initial treatment of PEA.

2020

73
Q

A 25-year-old man is being considered as an organ donor after sustaining traumatic brain injury in a motorcycle crash. Which of the following findings is most likely to confirm brain death?

A) Absence of deep tendon reflexes
B) Mild cough with tracheal tube manipulation
C) 1 mm pupillary constriction when tested for pupillary reflex
D) No movement of lid or eyes when touching cornea
E) Nystagmus with caloric testing

A

The correct response is Option D.

Brain death is a criterion for nonliving organ donation. Absence of all brain stem reflexes needs to be present for brain death to occur. Absence of corneal reflex demonstrates an absence of brain stem reflexes. Presence of pupillary reflex and nystagmus during caloric test demonstrates a positive finding and presence of brain stem reflex. Mild cough or gag during tracheal manipulation demonstrates presence of brain stem reflex. Any respiratory rate during apnea test is seen as a sign of brain stem function. Evaluation of deep tendon reflexes is not part of assessing brain death.

2020

74
Q

A 9-month-old female infant with severe metopic craniosynostosis requires bifrontal craniotomy and a fronto-orbital advancement. The anesthesiologist is concerned about intraoperative blood loss and the need for blood transfusion. Which of the following drugs administered by intravenous infusion at the time of cranial reconstruction is most likely to reduce both intraoperative and postoperative bleeding?

A) Aprotinin
B) Erythropoietin
C) Fibrinogen
D) Protamine
E) Tranexamic acid

A

The correct response is Option E.

Acute blood loss and the need for autologous blood transfusions are common in infants undergoing craniofacial procedures. Techniques to limit blood loss and transfusions have been studied extensively in craniosynostosis surgery. Tranexamic acid (TXA) is a synthetic analog of the amino acid lysine, which inhibits the conversion of plasminogen to plasmin when intravenously administered. TXA inhibits the proteolytic action of plasmin, thus inhibiting fibrinolysis. It has been shown in multiple studies to reduce both intraoperative and postoperative blood loss.

Aprotinin is a small protein bovine pancreatic trypsin inhibitor with anti-thrombolytic potential. It was taken off the market in the United States in 2007 as its use was associated with increased complications and death. It has since been reapproved in Canada and Europe but remains banned in the United States. Protamine is a drug used to reduce the effects of heparin toxicity and of no benefit in this clinical scenario. Erythropoietin is a recombinant glycoprotein cytokine that stimulates red cell production. When given preoperatively, it has been shown to reduce the need for blood transfusion (not necessarily acute blood loss) in craniosynostosis surgery. Fibrinogen is not administered intravenously. Fibrin glue does reduce blood loss in craniosynostosis procedures, but it is administered topically, not intravenously.

2020

75
Q

A 48-year-old man who sustained severe traumatic brain injury and extensive facial fractures in a motor vehicle collision 2 weeks ago is receiving ventilatory support and enteral nutrition via feeding tube. An indirect calorimetry assessment shows a respiratory quotient (RQ) value of 1.3. This value is most closely associated with which of the following types of metabolism?

A) Carbohydrate oxidation
B) Ketosis
C) Lipid oxidation
D) Lipogenesis
E) Protein oxidation

A

The correct response is Option D.

Indirect calorimetry measurements, or a metabolic cart, are used to help calculate the resting energy expenditure (REE) and respiratory quotient (RQ) as a means to help determine the caloric needs of a patient as well as optimize the patient’s nutrition. Indirect calorimetry measures the amount of oxygen consumed (VO2) and the amount of carbon dioxide produced (VCO2) by the patient. REE is calculated by the Weir equation [REE = (3.94 x VO2) + (1.1 x VCO2)] while the RQ is calculated as VCO2 / VO2. The calculated values of the respiratory quotient are reflections of what fuels are being oxidized by the patient with typical values as follows:

Optimal values of RQ for nutrition assessment are between 0.8 and 0.9, representing a balance between lipid and glucose oxidation. Values below 0.8 suggest underfeeding, while values greater than 1.0 suggest overfeeding. A value of 1.3 in this patient suggests significant overfeeding and lipogenesis and the enteral nutrition should be adjusted based on the current energy expenditure obtained from indirect calorimetry.

Ketosis is not measured by RQ.

2020

76
Q

A 37-year-old woman with macrotextured saline implants placed 10 years ago was recently diagnosed with a peri-implant fluid collection. Ultrasound-guided aspiration of the fluid is performed and sent for Wright-Giemsa–stained smears, and cell block immunohistochemistry/flow cytometry testing. Images are shown. Which of the following results would confirm a diagnosis of breast-implant–associated anaplastic large cell lymphoma?

A) CD30 negative, ALK negative
B) CD30 negative, ALK positive
C) CD30 positive, ALK negative
D) CD30 positive, ALK positive

A

The correct response is Option C.

This patient has breast implant–associated anaplastic large cell lymphoma (BIA-ALCL) which is a distinct form of CD30-positive T-cell, non-Hodgkin’s lymphoma that arises in association with a breast implant after either reconstructive or cosmetic surgery. The disease is typically contained within the capsule and fluid immediately adjacent to the implant. Patients commonly present with delayed seroma, but can also present with pain, capsular contracture, and/or a palpable mass. BIA-ALCL risk is higher with textured devices and these concerns led to the 2019 FDA recall of Allergan Biocell devices. The recall includes older McGhan and Inamed implants and current Natrelle implants. When patients with suspicious history or symptoms are evaluated, pre-operative imaging (e.g., mammography, ultrasound, and/or MRI) is recommended with aspiration of identifiable fluid or biopsy of mass. Ultrasound is considered the diagnostic modality of choice. Diagnostic evaluation should indicate concern for BIA-ALCL to the pathologist to include cytological evaluation of seroma fluid or mass with Wright-Giemsa stained smears and cell block immunohistochemistry/flow cytometry testing for cluster of differentiation (CD30) and anaplastic lymphoma kinase (ALK) markers. Wright-Giemsa staining shows pleomorphic cells with horseshoe shaped nuclei, nuclear folding and abundant vacuolated cytoplasm. All known cases of BIA-ALCL are CD30-positive and -negative for ALK, distinct from systemic ALCL which is ALK-positive.

2021

77
Q

A 15-year-old girl presents for evaluation of hypoplasia of the right breast. Examination shows an underdeveloped left breast, superiorly displaced nipple-areolar complex, and sunken-appearing chest wall. The pectoralis major muscle and sternum are normal. Which of the following is the most likely diagnosis?

A) Amastia
B) Amazia
C) Anterior thoracic hypoplasia
D) Athelia
E) Poland syndrome

A

The correct response is Option C.

Anterior thoracic hypoplasia is characterized by hypoplasia of the breast in the context of normal sternum and normal pectoralis major muscle. It is on the differential diagnosis of congenital breast deformities, and is distinguished from Poland syndrome by the normal pectoralis major muscle. Pectus excavatum is another condition of the chest wall; it is characterized by abnormal development of the sternum and ribs, and does not affect breast growth, although it can cause medial displacement of the breasts. Pectus excavatum is more common in males than females. Treatment of the breast in Poland syndrome and anterior thoracic hypoplasia depends on the degree of deformity and the goals of the patient; both implant-based reconstruction and fat grafting have been used.

Amazia is an absence of the mammary gland with a present nipple areolar complex, and amastia is complete absence of the mammary gland and nipple areolar complex.

Athelia is an absence of the nipple.

2021

78
Q

A 34-year-old woman desires improvement of the appearance of her breasts and abdomen. Physical examination shows a supernumerary nipple with a small bud of breast tissue just under the fold of the left breast. The embryologic origin and development of this accessory structure occur as a result of which of the following?

A) Incomplete differentiation of the ectodermal ridge
B) Incomplete differentiation of the mesodermal ridge
C) Incomplete involution of the ectodermal ridge
D) Incomplete involution of the mesodermal ridge

A

The correct response is Option C.

In utero, the breasts develop from paired mammary ridges of thickened ectoderm which extend from the axillae to the inguinal regions. The ectoderm will give rise to the nipple and ductal elements, while the mesoderm will eventually give rise to the connective tissue and vascular structures of the breast. Polythelia, or supernumerary nipples, occur as a result of incomplete involution of one of the many epithelial buds along the mammary ridge.

2021

79
Q

A 20-year-old woman presents with concerns about the appearance of her breasts. Examination shows unilateral herniation of the nipple-areola complex and a constricted lower pole. Which of the following maneuvers is most likely to address this patient’s concerns?

A) Elevation of the inframammary fold
B) Radial release of parenchymal bands
C) Skin grafting of nipple-areola complex
D) Vertical mastopexy
E) Wise pattern reduction mammaplasty

A

The correct response is Option B.

The patient description is consistent with tuberous breast deformity. Surgical intervention usually includes reduction of the periareolar herniation with periareolar incisions and radial release of parenchymal bands. Reduction would not address the issues associated with a tuberous breast. Vertical mastopexy alone through standard approaches would potentially further constrict the lower pole. Skin grafting of the nipple-areola complex would not address the tissue herniation. Elevation of the inframammary fold would not address the tuberous deformity and might exacerbate it.

2021

80
Q

An otherwise healthy, 24-year-old woman presents for breast augmentation consultation. Physical examination shows polythelia. On the basis of this finding, which system is most likely to have associated abnormalities?

A) Gastrointestinal
B) Hematological
C) Pulmonary
D) Renal
E) Vertebral

A

The correct response is Option D.

The presence of a supernumerary nipple occurs in 2-6% of females. Polythelia is the presence of two or more supernumerary nipples. A correlation exists between renal disease and polythelia. It is associated with 19% of patients with renal adenocarcinoma and 16.5% of patients with end-stage renal disease. Regular physical examination and urinalysis should be performed in patients with polythelia and any noted abnormality should alert the physician to the need for a renal ultrasound.

2021

81
Q

A 19-year old woman presents with concerns about the appearance of her breasts. Examination shows asymmetrical breasts with grade III ptosis, small breast footprint of both breasts, and a large nipple-areola complex with herniation of breast tissue. Which of the following characteristics of the tuberous breast deformity contributes to the physical findings in this patient?

A) Hyperplasia of one or more quadrants
B) Hyperplasia of only medial quadrants of the breast
C) Hypoplastic areola
D) Low inframammary fold
E) Periareolar ring constriction

A

The correct response is Option E.

Tubular breasts are caused by connective tissue malformations and occur in puberty. Clinical characteristics include breast asymmetry, dense fibrous rings around the areola, hernia bulging of the areola due to hypoplastic fascial support, hypoplasia of one, two or more quadrants, narrowing of the breast base, and a high location of submammary folds.

2021

82
Q

A 32-year-old woman comes to the office 2 years after undergoing bilateral breast augmentation with 350-cc smooth, round saline implants. The patient reports chronic fatigue and joint pain. Physical examination shows soft, symmetrical, and nontender breasts. There are no palpable masses and no axillary adenopathy. Which of the following is most appropriate next step in management?

A) Exchange saline implants for cohesive gel implants
B) Order MRI
C) Perform en bloc removal of the implants
D) Perform stereotactic biopsy of capsule
E) Request autoimmune disease evaluation

A

The correct response is Option E.

Breast implant illness (BII) is a term used to describe a multitude of symptoms seen in patients with breast implants. These symptoms may include the following (as well as others): fatigue, anxiety, headaches, brain fog, anxiety, photosensitivity, hormonal issues, rash, and hair loss. There is no definitive link between these symptoms and breast implants. Research is ongoing.

It is important that all patients with these symptoms are evaluated. BII has been self reported by patients with all types of implant characteristics including silicone, saline, textured surfaces, and smooth-walled implants.

Before undergoing surgery, evaluation of these patients is important. Many of these symptoms can be associated with known autoimmune diseases and should be evaluated for this possibility, either by a rheumatologist or other medical professional. If a known autoimmune disease is diagnosed, then traditional treatment for this known disease should be tried before explantation surgery.

Women who underwent explantation for possible BII had varying degrees of improvement including no improvement, temporary improvement, and permanent resolution of symptoms. In one study, patients with documented autoimmune disease showed no improvement following explantation.

MRI would not initially be needed with a normal physical examination and saline implants.

Stereotactic biopsy would be indicated for palpable masses or breast mass evaluation, not symptoms of BII.

2021

83
Q

A 19-year-old man sustains a flexor tendon laceration while cutting an avocado. He presents to the emergency department and subsequently undergoes flexor tendon repair surgery. He is prescribed opioids for post-surgical pain management. Which one of the following has been shown to be a risk factor for prolonged opioid use?

A) Male gender
B) Tendon surgery
C) Trauma surgery
D) Younger age

A

The correct response is Option D.

In the realm of upper extremity surgery, younger age, bone procedures, mental health disorders, history of substance abuse, and pain disorders have consistently been found to be risk factors for prolonged opioid use. A specific gender has not been consistently shown to be a risk factor of opioid abuse following upper extremity surgery, with some studies pointing to female gender, while others point to male gender. Other factors that have been identified in specific studies include elective versus traumatic hand surgeries, lower versus higher income, and comprehensive insurance coverage versus medicare insurance. Aside from identifying risk factors, the majority of recent studies regarding prolonged opioid use after either upper extremity surgery or plastic surgery point to overprescription of opioids as a common problem. Recent reports demonstrate that narcotics are overprescribed by 40 to 60% after upper extremity surgery.

2021

84
Q

A 42-year-old woman undergoes wide local excision and sentinel node biopsy of an invasive melanoma of the forearm. After uneventful induction of general anesthesia, 1 mL of isosulfan blue dye is injected intradermally around the healing biopsy site on the forearm. The operation begins with excision of the axillary sentinel lymph node, which is identified high in the axilla. During removal of the sentinel node, blood pressure decreases to 60/40 mmHg. After discontinuing the procedure and administering a bolus of intravenous fluid, which of the following is the most appropriate next step?

A) Administer dexamethasone
B) Administer diphenhydramine
C) Administer lipid emulsion
D) Administer phenylephrine

A

The correct response is Option D.

Isosulfan blue has many uses. It is used to identify sentinel nodes in melanoma and Merkel cell skin cancer as well as in breast cancer. It is used in reverse axillary mapping in order to preserve extremity lymph nodes while harvesting those that drain the breast. Allergic reactions to this dye occur in up to 1.6% of patients.

These adverse events are unexpected and occur with unpredictable severity. There is no current validated method to detect or decrease the risk of allergic reaction. It is important for the plastic surgeon to be able to quickly recognize and treat complications. The most common allergic reactions are urticaria, blue hives, and skin rash. Anaphylaxis has been reported, but much less commonly.

In this scenario, the first sign of anaphylactic reaction is the sudden and severe drop in blood pressure. This can occur immediately or unexpectedly later during the surgical procedure. After halting the operation and starting a fluid bolus, a vasopressor should be administered to counteract the hypotension. Once this is done, both a corticosteroid (dexamethasone) and an antihistamine (diphenhydramine) should be given to counteract the allergic reaction. Changing the fraction of inspired oxygen will not help the allergic reaction. A needle thoracostomy would be indicated if dissection high in the axilla resulted in a tension pneumothorax, but that is not the case here. Administration of lipid emulsion is appropriate for local anesthetic toxicity.

A commonly suggested alternative to isosulfan blue, without the risk of anaphylaxis, is methylene blue. However, there is a significantly higher rate of wound healing complications with the use of methylene blue, which is why isosulfan blue is preferred in plastic surgical procedures, specifically skin grafting.

2021

85
Q

A 43-year-old woman is scheduled to undergo bilateral mastectomies and immediate tissue-expander–based reconstruction for breast cancer. To decrease the postoperative pain and requirement for narcotics, preoperative ultrasound-guided pectoral nerve (PECS) 1 and 2 regional blocks with liposomal bupivacaine are planned. The PECS 1 block is administered. Which of the following is the most appropriate location for infiltration of the local anesthetic when performing the PECS 2 block?

A) Between the breast parenchyma and pectoralis major
B) Between the pectoralis major and pectoralis minor
C) Between the pectoralis minor and the serratus anterior
D) Between the serratus anterior and the latissimus dorsi

A

The correct response is Option C.

The pectoral nerve (PECS) 1 and PECS 2 anesthetic blocks have become increasingly common methods for regional anesthesia to decrease postoperative pain in many early recovery after surgery (ERAS) protocols. The PECS blocks are thoracic regional fascial plane blocks that typically require ultrasound guidance for proper infiltration of the anesthetic between the muscles of the thoracic wall.

A PECS 1 block is performed between the pectoralis major and pectoralis minor muscles and anesthetizes the lateral and medial pectoral nerves. A PECS 2 block, in addition to the PECS 1 block, involves an additional injection of anesthetic between the pectoralis minor and serratus anterior muscles and blocks the intercostal and intercostobrachial nerves. The serratus plane block is another thoracic regional fascial plane block requiring injection of local anesthetic between the serratus anterior and latissimus dorsi muscles in order to block the long thoracic and thoracodorsal nerves.

2021

86
Q

A 165.3-lb (75-kg), 76-year-old woman is scheduled to undergo wide local excision of a large, invasive basal cell carcinoma of the cheek with flap reconstruction during general anesthesia. Medical history includes nonvalvular atrial fibrillation, hypertension, and an embolic stroke 3 months ago. Current medications include warfarin. Renal function is normal. Which of the following is the most appropriate preoperative anticoagulation management for this patient?

A) Discontinue warfarin 5 days prior to the procedure and initiate low-molecular-weight heparin bridging 3 days prior to the procedure
B) Discontinue warfarin 5 days prior to the procedure without bridging
C) Discontinue warfarin 7 days prior to the procedure and initiate low-molecular-weight-heparin bridging 3 days prior to the procedure
D) Immediately initiate low-molecular-weight heparin bridging and discontinue warfarin 5 days prior to the procedure
E) Do not discontinue warfarin

A

The correct response is Option A.

This patient has a very high thromboembolic risk and a high bleeding risk. Recommended heparin bridging is 3 days before a planned procedure (ie, two days after discontinuing warfarin), when the prothrombin time and international normalized ratio (PT/INR) has started to drop below the therapeutic range.

Atrial fibrillation accounts for the highest percentage of patients for whom perioperative anticoagulation questions arise. Importantly, patients with atrial fibrillation are a heterogeneous group; risk can be further classified according to clinical variables such as age, hypertension, congestive heart failure, diabetes, prior stroke, and other vascular disease.

Bridging anticoagulation may be appropriate in patients who will have a very high thromboembolic risk with prolonged interruption of their anticoagulant (generally a vitamin K antagonist). Individual patient comorbidities that increase bleeding risk may also need to be considered because an increased postoperative bleeding risk may be a reason to avoid bridging. Suggested use of bridging in individuals taking warfarin includes:

Embolic stroke or systemic embolic event within the previous three months

Mechanical mitral valve

Mechanical aortic valve and additional stroke risk factors

Atrial fibrillation and very high risk of stroke (eg, systemic embolism within the previous 12 weeks, concomitant rheumatic valvular heart disease with mitral stenosis)

Venous thromboembolism (VTE) within the previous three months (preoperative and postoperative bridging)

Recent coronary stenting (e.g. within the previous 12 weeks)

Previous thromboembolism during interruption of chronic anticoagulation

The other answer choices are not appropriate strategies for this patient.

2021

87
Q

An otherwise healthy 60-year-old woman underwent breast reconstruction with right free transverse rectus abdominis musculocutaneous (TRAM) flap 1 day ago. Cardiac monitoring shows no P waves and an irregular QRS complex. The patient is asymptomatic. Blood pressure is 120/80 mmHg and heart rate is between 130 and 139 bpm. Which of the following is the most appropriate first-line therapy for this patient?

A) Amiodarone
B) Digoxin
C) Diltiazem
D) Metoprolol
E) Propafenone

A

The correct response is Option D.

Atrial fibrillation manifests as irregularly irregular QRS complexes without P waves on ECG. Postoperative atrial fibrillation (POAF) is multifactorial in origin, and occurs in 5 to 10% of patients undergoing non-cardiothoracic surgery. In the largest trial to date comparing rate versus rhythm control (Atrial Fibrillation Follow-Up Investigation of Rhythm Management [AFFIRM]), rhythm control was associated with a greater number of hospitalizations, torsades de pointes, pulmonary events, gastrointestinal events, bradycardia, and QT prolongation events. Rate control is the treatment strategy of choice, with the goal of 80 to 100 bpm. Metoprolol is the preferred beta blocking agent due to its efficient conversion between IV and oral routes, low cost, and clinician familiarity. Diltiazem (non-dihydropyridine calcium channel blocker) is a second-line therapy, and is intended for use if first-line therapy is ineffective at rate control at maximum doses or the first-line therapy is contraindicated. Digoxin is considered when other options are ineffective or contraindicated because of its narrow therapeutic window. Amiodarone has both beta-blocking and calcium channel blocking properties in addition to its antiarrythmic effects. Amiodarone can be used in patients with decreased ejection fractions. It is associated with acute pulmonary toxicity.

2021

88
Q

A 63-year-old man undergoes ventral hernia repair with component separation. On the third postoperative day, his serum potassium level is 6.7 mEq/L. Vital signs are stable. Electrocardiography discloses sinus rhythm with flattened P waves and peaked T waves. Initial therapy should include the administration of which of the following drugs?

A) Albuterol
B) Calcium gluconate
C) Dextrose and insulin
D) Furosemide
E) Sodium bicarbonate

A

The correct response is Option B.

Intravenous calcium gluconate should be given to this patient with severe hyperkalemia and associated electrocardiographic changes.

Hyperkalemia causes a decrease in the resting membrane potential, leading to increased myocardial excitability and cardiac arrhythmias, including ventricular fibrillation and asystole. Electrocardiographic changes associated with progressive hyperkalemia include peaked T waves, prolonged P-R segment, flattening/loss of P waves, widening of QRS complex, ectopic beats, ventricular fibrillation, conduction blocks, and asystole.

Neither a specific serum potassium level threshold nor an electrocardiographic pattern that predisposes patients to life-threatening cardiac arrhythmias has been well established. However, the initial therapy for patients presenting with a serum potassium level greater than 6 mEq/L and hyperkalemia-related electrocardiographic changes should focus on stabilizing the myocardium to prevent or reverse cardiac arrhythmias by intravenous administration of a calcium salt (gluconate or chloride). The onset of action is nearly immediate, but the duration of the protective effect is only 30 to 60 minutes. Therefore, repeat administration may be required.

Interventions to shift potassium intracellularly (intravenous dextrose and insulin, with or without nebulized albuterol/beta-2 agonist; intravenous sodium bicarbonate) or eliminate it from the body (intravenous furosemide, rectal or oral potassium-binding agents, hemodialysis) should be carried out as soon as possible after intravenous administration of calcium.

2021

89
Q

A 71-year-old man with chronic obstructive pulmonary disease (COPD) is transferred to the hospital with multiple facial lacerations and fractures following a fall. He has increasing confusion on arrival at the hospital. His wife mentions that he had bled a significant amount after the fall. Laboratory studies show:

On the basis of these findings, this patient’s primary metabolic disorder is which of the following?

A) Metabolic acidosis due to exacerbation of his COPD
B) Metabolic acidosis due to hemodilution from bleeding
C) Metabolic alkalosis due to exacerbation of his COPD
D) Metabolic alkalosis due to hemodilution from bleeding

A

The correct response is Option A.

The pH is low, indicating an acidosis, while the bicarbonate is high, indicating that a respiratory acidosis with metabolic compensation is present. The pH is low so the primary problem is an acidosis and is likely to be respiratory in nature. The PaCO2 is very high and indicates a respiratory acidosis is present. The very high PaCO2 level seen here is typical of a person with respiratory disease that results in retention of CO2, (ie, the primary clinical problem is respiratory failure due to chronic obstructive pulmonary disease).The most likely cause for this acid-base abnormality is an acute exacerbation of chronic obstructive pulmonary disease.

2021

90
Q

A 65-year-old man presents to the office for panniculectomy evaluation. Medical history includes anxiety, controlled hypertension, hyperlipidemia, and borderline diabetes that is controlled by diet only. Current medications include lisinopril, carvedilol, atorvastatin, zolpidem, and sertraline. After consultation, the surgeon determines the patient is a good candidate for panniculectomy. It is most appropriate for the patient to stop taking which of the following medications the night before surgery?

A) Atorvastatin
B) Carvedilol
C) Lisinopril
D) Sertraline
E) Zolpidem

A

The correct response is Option C.

All the medications listed are safe for use the night before surgery with the exception of an ACE inhibitor. While there have been some questions in the past regarding the safety of selective serotonin reuptake inhibitors (SSRIs) like sertraline due to bleeding concern, the current consensus is that it is safe to take.1 Beta blockers can have a beneficial effect for the cardiovascular system before surgery and thus, patients may take them even the morning of surgery.2 Lisinopril, however, can lead to hypotension during surgery and anesthetic management, and should be stopped the night before surgery. As an ACE inhibitor, it may counter the medications the anesthesiologist uses for blood pressure control.

2021

91
Q

A 67-year-old woman is scheduled to undergo carpal tunnel release under monitored anesthesia care with local anesthetic. Her comorbidities include type 1 diabetes mellitus (recent hemoglobin A1c is 7.4%), current everyday smoking, and rheumatoid arthritis, for which she takes methotrexate. Which of the following is the most appropriate antibiotic therapy to reduce this patient’s risk for surgical site infection?

A) Intravenous cefazolin, 30 minutes prior to induction, cephalexin 500 mg four times a day for 7 days postoperatively
B) Intravenous cefazolin, 30 minutes prior to induction, with no postoperative therapy
C) Intravenous cefazolin, 60 minutes prior to induction, cephalexin 500 mg four times a day for 7 days postoperatively
D) Intravenous cefazolin, 60 minutes prior to induction, with no postoperative therapy
E) No antibiotics indicated

A

The correct response is Option E.

Several studies have shown that timing of antibiotic delivery does impact the rate of surgical site infection (SSI). Current recommendations suggest that, when indicated, antibiotics should be given between 1 and 2 hours before surgery. There are no data to support a reduction in SSI risk when antibiotics are given within 30 minutes of surgery.

In the retrospective review by Bykowski et al, authors reported the SSI rates in patients undergoing elective soft-tissue surgery. Rates of SSI did not differ between patients who received preoperative antibiotics and those who did not (0.54% versus 0.26%, p less than 0.05). When a subgroup analysis was performed, SSI infection rates for patients who were active smokers, diabetics, and with procedure length greater than 60 minutes showed no difference with or without antibiotic administration.

Traditionally, patients with rheumatoid arthritis are viewed to be at higher risk for infection following surgery. There are no data to suggest that there is a risk reduction with the use of preoperative and/or postoperative antibiotics. Also, methotrexate does not increase one’s SSI risk and is therefore not an indication for perioperative antibiotic therapy.

2021

92
Q

An otherwise healthy 35-year-old woman is scheduled to undergo routine full abdominoplasty while receiving conscious sedation and local anesthesia. Which of the following intravenous sedation regimens is most likely to reduce this patient’s pain and anxiety while minimizing the risk for respiratory depression?

A) Fentanyl alone
B) Ketamine and fentanyl
C) Midazolam alone
D) Midazolam and fentanyl
E) Midazolam and propofol

A

The correct response is Option D.

The advantage of using this combination is that midazolam has excellent anxiolytic and amnestic effects, whereas fentanyl is an excellent short-acting analgesic. A recent multicenter, randomized study demonstrated that the combination of fentanyl and midazolam is superior to midazolam alone in decreasing the patient’s subjective report of pain and anxiety.

The main drawback of fentanyl is respiratory depression; however, it does have a very short half-life. Midazolam, in contrast, has minimal effects on the respiratory system except in some older patients, in whom lower doses must be used. Continuous oxygen saturation monitoring and checking the patient’s respiratory status and other vital signs every 5 minutes are important for patient safety. Medications are only administered in small doses at each 5-minute interval (no more than 50 mg of fentanyl and 2 mg of midazolam at a time). This helps achieve a steady-state effect. Both of these medications have antagonists that are able to reverse their effects. Flumazenil and naloxone, the antagonists of midazolam and fentanyl, respectively, should be readily available in the operating room. The surgeon should be familiar with their dosage and administration.

Ketamine would not resolve anxiety in this patient.

Use of propofol in combination with an opiate and benzodiazepine can be used for conscious sedation; however, the disadvantage of this combination is the higher risk of respiratory depression and the lack of a reversing agent for propofol. Because a deeper level of sedation can be maintained, this technique is preferable for selected patients who are very anxious. A recently published series of abdominoplasty with sedation using propofol used monitored anesthesia care by an anesthesiologist or nurse anesthetist.

Propofol and benzodiazepines have no significant analgesic effect.

2021

93
Q

A 26-year-old man is scheduled to undergo septorhinoplasty following a nasal bone fracture 3 years ago. He has mild von Willebrand disease. The day of the operation, the surgeon administers 0.3 ?g/kg of 1-deamino-8-D-arginine-vasopressin before and after the surgery to help decrease postoperative bleeding. Which of the following is the initial mechanism by which the administered medication facilitates hemostasis in this patient?

A) Activating the Factor V Leiden molecule in the clotting cascade
B) Cleaving the fibrinogen molecule to fibrin
C) Inducing the release of von Willebrand factor from its storage sites in endothelial cells
D) Irreversibly blocks the formation of thromboxane A2 in platelets
E) Supporting complex formation with tissue factor, thereby providing enough thrombin to form fibrin plugs to stop minor bleeds

A

The correct response is Option C.

This patient is suffering from a bleeding disorder called von Willebrand disease (VWD), which occurs when the von Willebrand factor (VWF) is deficient or qualitatively abnormal.

Von Willebrand factor (VWF) works by mediating the adherence of platelets to one another and to sites of vascular damage. It also binds to Factor VIII, keeping it inactive while in circulation since Factor VIII rapidly degrades when not bound to VWF.

VWD is the most common of the inherited bleeding disorders, with an estimated prevalence in the general population of 1 percent by laboratory testing. Patients will often present with signs of easy bruising, extensive bleeding after dental work, heavy or long menstrual periods, and prolonged nose bleeds. Patients with a history of abnormal bleeding should always be properly worked up prior to surgery. There has been an extensive discussion over the years about the treatment of patients suffering from von Willebrand’s disease and rhinoplasty procedure.

The recommended treatment of patients suffering from VWD undergoing rhinoplasty procedure is 0.3 ?g/kg of 1-deamino-8-D-arginine-vasopressin (Desmopressin). Desmopressin is an analogue of vasopressin that exerts a substantial hemostatic effect, by inducing the release of von Willebrand factor from its storage sites in endothelial cells. Patients with the mild form of VWD have lower than normal levels of VWF, and the release of the additional proteins from the endothelial cells aids with clotting.

Factor V Leiden is a mutated form of human Factor V, which condition that result in a hypercoagulable state. Due to this mutation, Protein C, an anticoagulant protein which normally inhibits the pro-clotting activity of factor V, is not able to bind normally to Factor V, leading to a hypercoagulable state. Desmopressin does not have a direct interaction on this molecule.

Thrombin is an enzyme that converts fibrinogen to fibrin, and a reaction that leads to the formation of a fibrin clot. There are several thrombin products commercially available.

Hemophilia is a condition that is deficient in Factor VII. Recombinant activated factor VII.

Aspirin irreversibly blocks the formation of thromboxane A2 in platelets, producing an inhibitory effect on platelet aggregation.

2021

94
Q

A 38-year-old woman is undergoing a routine abdominoplasty at an outpatient surgery center under general anesthesia. Thirty minutes into the operation, the anesthesiologist reports high end-tidal CO2 production and tachycardia. Which of the following is the most appropriate first step in management?

A) Administer dantrolene
B) Discontinue volatile anesthetic agents
C) Infusion of lipid emulsion
D) Switch to total intravenous anesthesia
E) Treatment of arrhythmia

A

The correct response is Option B.

Malignant hyperthermia is an anesthetic crisis that is potentially fatal if not appropriately managed. The mechanism of malignant hyperthermia is an accelerated release of calcium from the sarcoplasmic reticulum. The increasing release of calcium surpasses uptake and leads to an inability to control the intracellular calcium level. The symptoms of malignant hyperthermia include unexplained high end-tidal CO2 levels, tachycardia, increasing body temperature, masseter muscle rigidity, and skeletal muscle rigidity. The first step in the management of malignant hyperthermia is discontinuing the volatile anesthesia. The subsequent steps in management include administering dantrolene, switching to IV anesthesia, and treatment of arrhythmia. Depolarizing muscle relaxants can cause malignant hyperthermia and these drugs should be immediately discontinued and not administered. Infusion of lipid emulsion is done for lidocaine toxicity.

2021

95
Q
A

The correct response is Option B.

Gabapentin is a structural analogue of the neurotransmitter gamma-aminobutyric acid and high affinity to voltage-gated calcium channels at presynaptic terminals of hyperexcited neurons through reduction in depolarization-influx of calcium required for release of excitatory neurotransmitters, including glutamate, noradrenaline, dopamine, and serotonin. The exact nature of its analgesic effect is incompletely understood. Contemporary evidence demonstrates that perioperative administration in conjunction with an ERAS protocol is associated with a significant reduction in postoperative opioid use.

Lidocaine is an amide local anesthetic that decreases the excitation threshold of nociceptive afferent neurons by decreasing neuronal membrane permeability through voltage-gated sodium channels. The inhibition of neuronal depolarization results in blockade of pain transmission.

Oxycodone is a semi-synthetic opioid that is primarily the mu-opioid receptor agonist that opens calcium-dependent inward-rectifying potassium channels, which causes hyperpolarization and reduced excitability of neuronal ascending pain pathways.

Acetaminophen was previously believed to exert its analgesic effect by inhibiting cyclooxygenase enzymes 1 and 2. Recent investigations demonstrated that the main analgesic mechanism is the metabolite p-aminophenol, which crosses the blood-brain barrier and is converted to N-acylphenolamine (AM404). AM404 then acts on the transient receptor potential vanilloid subtype 1 (TRPV1) and cannabinoid 1 receptors in the brain and terminals of C-fibers in the spinal dorsal horn. Both the brain and spinal dorsal horn are critical to pain pathways and modulate nociceptive transmission. Therefore, acetaminophen induces analgesia by acting on the brain and spinal cord.

Ketamine is a noncompetitive antagonist to the N-methyl-d-aspartate (NMDA) receptors. The NMDA receptor is involved in the amplification of pain signals, central sensitization, and opioid tolerance. The ketamine molecule is hydrophilic and lipophilic, allowing it to cross the blood-brain barrier to exert its NMDA antagonist effects in the brain and spinal cord.

2022

96
Q

A 35-year-old woman is interested in a tummy tuck and liposuction of the flanks and mid back. She does not smoke cigarettes, can climb two flights of stairs without shortness of breath or chest pain, and takes medication for hypothyroidism, which is well-controlled. Height is 5 ft 6 in (167 cm) and weight is 260 lbs (118 kg). BMI is 42 kg/m2. She is otherwise healthy. On the basis of this patient’s history, which of the following is the most appropriate American Society of Anesthesiologists (ASA) physical status classification?

A) ASA I
B) ASA II
C) ASA III
D) ASA IV
E) ASA V

A

The correct response is Option C.

This patient is classified as ASA III, on the basis of a BMI greater than 40 kg/m2. Thus, she may not be a candidate for surgical facilities that are restricted to ASA I or II cases, like many office-based surgical facilities. Other examples of health factors that potentially classify a patient as ASA III (or higher) include: history of transient ischemic attack, stroke, poorly controlled diabetes mellitus or hypertension, chronic obstructive pulmonary disease, active hepatitis, alcohol dependence or abuse, implanted pacemaker, moderate decrease of ejection fraction, end-stage kidney disease undergoing regularly scheduled dialysis, percutaneous coronary angioplasty within 60 weeks, and history of myocardial infarction or stents for coronary artery disease more than 3 months ago.

2022

97
Q

A 42-year-old woman is scheduled for abdominoplasty, repair of diastasis recti abdominis, and incisional herniorrhaphy. At the start of the operation, 20 mL of 1% lidocaine with epinephrine 1:100,000 is injected into the lower abdominal incision. After incisional herniorrhaphy is completed, the general surgeon plans to inject liposomal bupivacaine into the lower abdominal hernia repair. Which of the following is the most appropriate recommendation regarding the injection of liposomal bupivacaine after lidocaine injection?

A) Do not use liposomal bupivacaine after lidocaine is used
B) Wait at least 20 minutes before injecting liposomal bupivacaine
C) Wait at least 60 minutes before injecting liposomal bupivacaine
D) Wait at least 90 minutes before injecting liposomal bupivacaine
E) There are no restrictions

A

The correct response is Option B.

Wait at least 20 minutes before injecting liposomal bupivacaine. Liposomal bupivacaine (Exparel), is a long-acting (approximately 72 hours) local anesthetic that is an extended-release multivesicular liposomal version of bupivacaine. It has been shown that this drug can decrease the need for opiates in the postoperative period.

Nonbupivacaine anesthetics, including lidocaine, when mixed with liposomal bupivacaine may cause an immediate release of bupivacaine, potentially causing an overdose. For this reason, lidocaine and liposomal bupivacaine should not be mixed together. Administration of liposomal bupivacaine may follow the administration of lidocaine after a delay of 20 minutes or more. Administration of liposomal bupivacaine results in systemic plasma concentrations that can persist for 96 hours after local infiltration. That said, it is recommended that the use of local anesthetics be avoided within 96 hours following administration of liposomal bupivacaine. The other choices are incorrect because they do not conform to the recommendations and pharmacology of the involved medication.

2022

98
Q

According to the Malignant Hyperthermia Association of the United States guidelines, which of the following intravenous agents should be avoided during the acute resuscitation phase of malignant hyperthermia, following dantrolene administration, when treating subsequent cardiac dysrhythmias?

A) Dextrose 50% solution
B) Epinephrine
C) Lidocaine
D) Metoprolol
E) Verapamil

A

The correct response is Option E.

Malignant hyperthermia (MH) is a disturbance of calcium channel homeostasis, causing unregulated calcium release from sarcoplasmic reticulum. The coadministration of dantrolene used during resuscitation and an intravenous calcium channel blocker is contraindicated, since both verapamil and diltiazem have been associated with life-threatening hyperkalemia in this setting. The other agents listed (metoprolol, lidocaine, epinephrine, and dextrose 50%) may be used as needed during advanced cardiovascular life support protocols or for the treatment of hyperkalemia. MH is a pharmacogenetic disorder triggered in genetically susceptible individuals by volatile anesthetic gases and succinylcholine. It has autosomal dominant inheritance. Dantrolene is the only treatment for a MH crisis, and a newer formulation has faster administration than traditional preparations. The resuscitation poster for MH should be in every operating room facility and is mandated by the American Association for Accreditation of Ambulatory Surgical Facilities (AAAASF) standards. Their phone number is 1-800-MH-HYPER.

2022

99
Q

A 42-year-old man presents with partial degloving injury of the right leg and fracture of the pelvis sustained during a motorcycle collision 2 weeks ago. Medical history includes factor V Leiden. BMI is 31.1 kg/m2. Skin graft reconstruction of the degloving injury of the leg is planned. According to the Caprini Risk Assessment Model (2005), which of the following conditions in this patient represents the greatest risk for perioperative venous thromboembolism?

A) Age
B) BMI greater than 25 kg/m2
C) Major surgery (greater than 45 minutes in duration)
D) Pelvic fracture
E) Positive factor V Leiden

A

The correct response is Option D.

The ASPS Venous Thromboembolism Task Force Report includes recommendations for risk stratification based on the Caprini Risk Assessment Model (RAM), which has been widely validated in the scientific literature for evaluation of surgical patients. The 2005 model in particular is most applicable to plastic surgery. This model awards a score based on various weighted patient risk factors, with venous thromboembolism (VTE) risk increasing as the score increases. The literature suggests that patients with a Caprini RAM score less than 8, who were not provided chemoprophylaxis, experienced VTE at a rate of 11.3%.

The Caprini RAM score for this patient is 12. Age 41 to 60 years is 1 point. BMI greater than 25 kg/m2 is 1 point. Major surgery (longer than 45 minutes) is 2 points. Positive factor V Leiden, an inheritable mutation in factor V protein, which is involved in the blood clotting cascade, is 3 points. Hip, pelvis, or leg fracture within the past month is 5 points. Based on this risk stratification tool, the surgeon “should strongly consider the option to use extended low molecular weight heparin postoperative prophylaxis” in this patient.

2022

100
Q

A 55-year-old man undergoing excision of a soft-tissue mass of the left thigh develops malignant hyperthermia shortly after induction of general anesthesia. The surgical procedure is promptly aborted, and the patient is successfully treated. Six months later, he returns for another attempt at excising the mass. The use of which of the following anesthetic agents is most appropriate in this case?

A) Ether
B) Halothane
C) Propofol
D) Sevoflurane
E) Succinylcholine

A

The correct response is Option C.

Lidocaine and all other local anesthetics are considered safe to be used in patients who are susceptible to malignant hyperthermia.

Volatile anesthetic agents (halothane, sevoflurane, ether) and depolarizing muscle relaxants (succinylcholine) are considered potential triggers of malignant hyperthermia and, therefore, should be avoided in susceptible individuals.

Other anesthetic agents that are considered safe to be used in patients susceptible to malignant hyperthermia include:

  • nonvolatile general anesthetics: nitrous oxide
  • opioids: morphine, fentanyl, alfentanil, hydromorphone, meperidine, naloxone
  • barbiturates and intravenous anesthetics: thiopental, propofol, etomidate
  • nondepolarizing muscle relaxants: pancuronium, atracurium, rocuronium
  • benzodiazepines: lorazepam (Ativan), midazolam (Versed), klonopin

Dozens of genetic mutations have been associated with susceptibility to malignant hyperthermia, which is generally inherited in an autosomal dominant pattern.

2022

101
Q

A 78-year-old man who was recently diagnosed with low rectal cancer presents to the office for evaluation. The patient is accompanied by his daughter, who fills out all the questionnaires for him. Medical history includes hypertension, osteoarthritis, and bilateral knee replacements 20 years ago. Vital signs are within normal range. BMI is 21.5 kg/m2. He ambulates with a walker. Physical examination is otherwise unremarkable. An abdominoperineal resection and perineal reconstruction are planned. Obtaining which of the following is the most appropriate next step in assessing this patient’s risk for perioperative complications?

A) APACHE II score
B) FEV1/FVC ratio
C) Frailty index
D) Liver function tests
E) Sequential Organ Failure Assessment (SOFA) score

A

The correct response is Option C.

Obtaining a frailty score would help providers assess the perioperative risk for this elderly patient with limited ambulation and possible cognitive impairment who is about to undergo a significant colorectal and reconstructive operation.

Frailty is a state of functional compromise in an elderly patient, which may include declined ability to maintain homeostasis, loss of physiological reserve, and increased vulnerability to adverse outcomes in the perioperative period.

Chronological age alone has been found to be a poor predictor of cancer and/or surgery tolerance by elderly patients. This subgroup of surgical patients has been rapidly increasing in number due to overall aging of the US population and can be quite heterogeneous when it comes to individual health status. Therefore, preoperative evaluation of elderly patients should include medical comorbidities and baseline functional status.

Several frailty scoring systems have been described, including the Phenotype Model, Cumulative Deficit Model, Comprehensive Geriatric Assessment, Risk Analysis Index, Charlson Comorbidity Index, Hopkins Score, and Frailty Index, along with its 11- and 5-item modified/simplified versions. Scored variables include physical characteristics (unintentional weight loss, slow gait, weak grip strength, etc.), medical history (stroke with deficit, myocardial infarction, congestive heart failure, etc.), and laboratory results (kidney function, nutrition, etc.).

Higher frailty scores have been associated with higher risk for in-hospital complications, longer hospital stay, discharge disposition to a skilled nursing facility, increased postoperative complications, and decreased postoperative survival at 30, 180, and 365 days.

Liver function may not be indicated in this patient without any history of abnormal bleeding or jaundice or signs of liver disease. FEV1 (forced expiratory volume) to FVC (forced vital capacity) ratio is used to monitor airway flow and assess response to therapy in patients with asthma. APACHE (Acute Physiology and Chronic Health Evaluation) II and SOFA (Sequential Organ Failure Assessment) are used to grade severity of illness in critically ill patients and predict clinical outcomes in the ICU setting.

2022

102
Q

A 22-lb (10-kg), 8-month-old infant is undergoing cranial vault reconstruction. The anesthesiologist informs the surgeon that the estimated blood loss is 150 mL. Approximately which of the following percentages of this patient’s circulating blood volume has been lost?

A) 10%
B) 20%
C) 30%
D) 40%
E) 50%

A

The correct response is Option B.

In infants, the formula used for estimated circulating blood volume is 75 to 80 mL per kg of body weight. This infant’s estimated circulating volume is 750 mL total. When performing surgery with blood loss that is large relative to circulating volume, blood loss control and blood replacement need to be watched carefully.

Estimated circulating blood volume in a 154-lb (70-kg) adult is 5.5 L. For neonates, the formula is increased to 85 to 90 mL/kg, and for children, the formula is decreased to 70 to 75 mL/kg. The surgeon should be aware when working on infants of the changing estimate of circulating blood volume based on weight. A reference for a recent nomogram for prediction of hemoglobin and hematocrit shifts after blood loss and resuscitation is included in the resources, as well as classic papers on fluids in pediatric patients and estimations of estimated circulating volume.

2022

103
Q

An otherwise healthy 45-year-old woman presents for abdominoplasty. Administration of liposomal bupivacaine for postoperative pain control is planned. Which of the following is the maximum dose of liposomal bupivacaine that can be administered in a single dose in this patient?

A) 50 mg
B) 133 mg
C) 200 mg
D) 266 mg
E) 399 mg

A

The correct response is Option D.

Liposomal bupivacaine is frequently used to manage postoperative pain. The mechanism of action is bupivacaine loads into multivesicular liposomes, thus allowing for slow release of bupivacaine. The potential efficacy of liposomal bupivacaine can last between 72 to 96 hours. Thus, studies have shown that administration of liposomal bupivacaine can decrease postoperative pain and also decrease the usage of opioids. The administration of liposomal bupivacaine is not weight-dependent, and the maximum dose recommended is a single administration of 266 mg or 20 mL. To cover a larger area, volume expansion with saline or bupivacaine can be used by dilution of the 266 mg; however, it is not recommended to exceed this dose.

2022

104
Q

A 63-year-old woman with a history of breast cancer presents for delayed reconstruction with bilateral latissimus dorsi muscle flaps. Medical history includes coronary artery disease, hypertension, and rheumatoid arthritis. Current medications include aspirin, propranolol, and prednisone, which she has taken for 8 years. In preparation for surgery, prednisone is discontinued 6 weeks before and aspirin is discontinued 1 week before surgery. The surgical procedure is uneventful. In the PACU, her heart rate is 115 bpm and blood pressure is 80/40 mmHg. Physical examination shows no signs of hematoma. A total of 3 L of intravenous fluid boluses are administered, with no hemodynamic improvement. Chest x-ray, ECG, hematocrit and serum electrolytes, and troponins are normal. Despite increasing doses of vasopressors during the next 2 hours, the patient remains hypotensive. Administration of which of the following is the most appropriate next step in management?

A) Dantrolene
B) Desmopressin (DDAVP)
C) Hydrocortisone
D) Insulin and dextrose
E) Labetalol

A

The correct response is Option C.

The most appropriate next step in managing this patient with otherwise unexplainable refractory hypotension and a recent history of long-term steroid use is intravenous administration of hydrocortisone, with the presumptive diagnosis of adrenal crisis.

The first case reports of surgery-induced acute adrenal insufficiency in patients on long-term steroid therapy were published in the early 1950s, just a few years after the release of oral cortisone in 1949. Today, over 6 decades later, significant controversy remains in the prevention, diagnosis, and treatment of this disease, despite its fatality if not appropriately prevented or managed.

Clinical signs of acute adrenal insufficiency, or adrenal crisis, include hypotension, hypoglycemia, dehydration, altered mental status, and hyponatremia, which can quickly progress to fatal hemodynamic collapse.

Prescribed steroid therapy (PST) can cause suppression of the hypothalamo–pituitary–adrenal (HPA) axis, placing these patients at risk for acute adrenal insufficiency as a consequence of surgical stress. Prednisone doses ? 5 mg/day (or hydrocortisone-equivalent dose) in adults via any route of administration (oral, inhaled, topical, intranasal, intra-articular) are sufficient to raise concern. There is no consensus on the minimal duration of PST to place patients at risk for an adrenal crisis, but 3 weeks is commonly cited. The risk may remain for up to 1 year after discontinuation of PST.

There seems to exist no consensus on the preoperative evaluation of patients at risk. Laboratory evaluation of their HPA axis is possible (e.g., ACTH challenge), but results correlate poorly with actual incidence of adrenal crisis. There is also no consensus on a prophylactic steroid regimen for patients at risk, although a recent trend away from the perioperative administration of high-dose steroids purely as prophylaxis can be observed, particularly in the inflammatory bowel disease literature.

Apart from all the controversy, at least a couple of principles related to this disease seem to be universally accepted: 1) a high degree of suspicion should be exercised when encountering unexplained refractory hemodynamic instability in a patient on PST subjected to stress by surgery or illness; 2) in such a case, preference should be given to prompt administration of rescue steroids, considering the expected mortality of untreated crises and the lack of evidence of long-term adverse consequences from short-term steroid administration.

None of the other drugs listed are directly pertinent to the management of the adrenal crisis in this patient. Desmopressin (DDAVP) causes the release of von Willebrand factor from platelets and endothelial cells. It is used in cases of factor VIII or von Willebrand factor deficiency. Insulin and dextrose solutions are used to shift potassium intracellularly in the treatment of hyperkalemia. Dantrolene is used in the treatment of malignant hyperthermia. Labetalol, a beta-adrenergic blocker, would probably worsen the patient’s hypotension.

2022

105
Q

Tranexamic acid inhibits which of the following enzymatic conversions?

A) Factor VII to factor VIIa
B) Factor X to factor Xa
C) Fibrinogen to fibrin
D) Plasminogen to plasmin
E) Prothrombin to thrombin

A

The correct response is Option D.

Tranexamic acid (TXA) has seen an expanding role in plastic surgery in recent years to decrease blood loss. Its mechanism of action is to inhibit the conversion of plasminogen to plasmin, which in turn inhibits the fibrinolytic pathway (clot degradation). All of the other options are steps involved in the clotting cascade, which are required for hemostasis (clot formation). TXA does not have an effect on any of those steps.

2022

106
Q

A 37-year-old woman undergoes breast augmentation and lipo-abdominoplasty with an overnight stay. She has no history of complicated or drug-resistant infections and no known drug allergies. To decrease the risk for surgical site infection, which of the following is the most appropriate antibiotic, dosing, and duration for this patient?

A) Cefazolin, first and only dose within 1 hour of incision
B) Cefazolin, first dose within 1 hour of surgery, continue for 23 hours postoperatively
C) Cefuroxime, first and only dose within 1 hour of incision
D) Cefuroxime, first dose within 1 hour of incision, continue for 23 hours postoperatively
E) No antibiotic therapy necessary

A

The correct response is Option A.

Due to growing concern over antibiotic misuse, patterns of antibiotic prescribing and adherence to guidelines must be surveyed. In 2003, Lyle et al. examined the prescribing practices of plastic surgeons with respect to different surgical procedures and compared with previous surveys. Authors found that surgeons were prescribing prophylactic antibiotics in more than 70% of cases for all procedures with the exception of blepharoplasty and chemical peel.

Though the evidence supports judicious antibiotic prescribing and consistently denounces long-term antibiotic therapy, surgeons often fail to adhere to these guidelines, in favor of longer duration antibiotic prophylaxis.

Ariyan et al. published an evidence-based consensus statement in 2015, detailing antibiotic prophylaxis to prevent surgical site infections (SSI) in plastic surgery. They performed a meta-analysis of the best available evidence across common plastic surgery procedures.

With respect to cosmetic breast and body surgery, authors analyzed data from 12 breast studies (three RCTs, nine non-RCTs) and 1 abdominal study (non-RCT). They concluded that the meta-analysis of the RCTs in clean breast surgery showed a significant reduction in risk for SSI with antibiotic prophylaxis versus control (2.5 vs 11.4%; OR, 0.16; 95% CT, 0.04 to 0.061; p = 0.01). When combined with non-RCTs, there was still a significant risk reduction (3.8 vs 6.7%; OR, 0.50; 95% CI, 0.26 to 0.94; p = 0.03).

With respect to abdominoplasty, the authors of the consensus statement reported the results of a pseudorandomized study in which antibiotic prophylaxis did not result in a significant reduction in SSI (6.5 vs 13.0%; OR 0.47; 95% CI, 0.18 to 1.23; p = 0.12).

Assuming that there is no contraindication, Cefazolin is generally recommended as a first-line, prophylactic antibiotic for clean plastic surgery procedures. CDC guidelines recommend antibiotic administration between 30 to 60 minutes from the initial incision. Two grams of cefazolin is indicated for patients less than 120 kg, and 3 grams is recommended for those over 120 kg.

Current data do not support prescribing additional antibiotics beyond a single preoperative dose.

2022

107
Q

A 24-year-old woman underwent uneventful cosmetic bilateral breast augmentation. One day postoperatively, the patient reports difficulty urinating with pubic fullness and discomfort. Which of the following is the most likely cause of this patient’s symptoms?

A) Acute tubular necrosis
B) Adverse effect of anesthesia
C) Inadequate resuscitation
D) Poor pain control
E) Urinary tract infection

A

The correct response is Option B.

Postoperative urinary retention (POUR) is a common postoperative complication that most often arises secondary to the general anesthesia given during surgery. The prevalence of POUR has been cited as up to 70% in the literature. The anesthetic medications used during the procedure can suppress micturition control and reflexes at the level of the central nervous system and peripheral nervous system while also acting as a smooth muscle relaxant that decreases bladder contractility. Management of POUR can be conservative, including ambulation and cessation of systemic analgesics, or invasive with either intermittent or indwelling catheter placement.

Correction of poor pain control with additional narcotic medications would likely worsen POUR. Urinary tract infection, while a possibility, would be associated with pain and burning during urination as opposed to pubic fullness and discomfort. Inadequate resuscitation would present with a lack of bladder fullness associated with oliguria. Acute tubular necrosis is associated with renal failure which this patient is unlikely to have.

2022

108
Q

A 57-year-old woman undergoes right mastectomy with immediate deep inferior epigastric perforator flap breast reconstruction for treatment of right breast cancer. BMI is 28 kg/m2. Preoperative Caprini RAM score is 6. Perioperative pneumatic compression boots and low-dose heparin are initiated. On postoperative day 2, the patient becomes tachycardic after walking to the bathroom. Resting heart rate is 110/bpm, blood pressure is 118/40 mm Hg, respiratory rate is 24/min, and pulse oximetry is 93% on 6 L nasal cannula oxygen supplementation. Urine output is 50 mL per hour over the past 8 hours. Which of the following is the most likely diagnosis?

A) Active bleeding
B) Anxiety
C) Caffeine withdrawal
D) Hypovolemia
E) Pulmonary embolism

A

The correct response is Option E.

The patient most likely has a pulmonary embolism. Pulmonary embolisms are rare but deadly complications following microsurgical breast reconstruction, and they often present with tachycardia alone (with or without hypoxia). The patient is receiving 6 L nasal cannula oxygen supplementation, so hypoxia is difficult to assess. Anxiety would commonly present as tachycardia with hypertension. Hypovolemia and active bleeding would commonly present with tachycardia with hypotension and decreased urine output. Caffeine withdrawal would commonly present with a headache.

2022

109
Q

A 61-year-old woman presents to discuss oncoplastic reduction mammaplasty. Medical history includes a femur fracture sustained during a fall 4 months ago. Recovery was complicated by a pulmonary embolus. Current medications include warfarin therapy. Her hematologist has recommended bridging therapy to prevent recurrent embolus. Which of the following is the most appropriate regimen to bridge anticoagulation with low-molecular weight heparin (LMWH) in this patient?

A

The correct response is Option D.

Managing anticoagulation in the perioperative period is challenging. The risk for thromboembolic phenomenon is transiently increased with any cessation in anticoagulation therapy. At the same time, this risk needs to be weighed against the risk for bleeding. For the patient described, who is high-risk for recurrent embolus (recent PE) and bleeding (due to the nature of the proposed procedure), these opposing risks need to be weighed carefully. They may be mitigated by a “bridging” strategy where short-acting agents provide prophylactic anticoagulation around the time of surgery while long-acting agents are held.

While the decision to adjust anticoagulation therapy in the perioperative period is controversial, the mechanism of bridging therapies is based on the known pharmacokinetic behaviors of anticoagulation agents. The half-life of warfarin is approximately 36 hours, so stopping warfarin prior to surgery will allow the therapeutic INR to return to normal levels by the day of surgery. To maintain therapeutic anticoagulation, low-molecular weight heparin (LMWH) may be initiated 3 days prior to surgery as the INR decreases. In the absence of other factors that predispose the patient to bleeding events, warfarin may be safely resumed 1 to 2 days after surgery.

Stopping warfarin 1 to 2 weeks prior to is unnecessary. Furthermore, a longer interval between cessation of warfarin and initiation of LMWH treatment may create a longer window where the patient is at risk for thromboembolic events.

Stopping warfarin the day of surgery and initiating LMWH the same day will place the patient at an increased risk for bleeding.

2022

110
Q

A 42-year-old woman undergoes deep inferior epigastric artery perforator flap breast reconstruction. An ultrasound-guided transversus abdominis plane regional block with liposomal bupivacaine is planned. The ultrasound probe is placed directly over the abdominal wall musculature in the anterior axillary line. The sonographic image is shown. Injection at which of the following levels is most appropriate?

A

The correct response is Option C.

Transversus abdominis plane regional anesthetic blocks have been shown to decrease narcotic consumption and abdominal pain following abdominal-based microsurgical breast reconstruction. The sensory nerves to the anterior abdominal wall run in the interfascial plane between the internal oblique and transversus abdominis muscles. A successful regional block depends on precise identification of the proper plane for the injection. The arrow indicated by letter C represents the transversus abdominis plane.

The arrow indicated by letter A indicates the interfascial plane between the internal and external oblique muscles.

The arrow indicated by letter B indicates the body of the internal oblique muscle.

The arrow indicated by letter D indicates the peritoneum.

The arrow indicated by letter E indicates the contents of the abdominal cavity.

2022

111
Q

A 65-year-old man undergoes hemimandibulectomy and reconstruction with a fibular flap for oral squamous cell carcinoma. On postoperative day 4, the patient develops a cough and feels ill. Temperature is 39.4°C (102°F), blood pressure is 80/40 mmHg, heart rate is 120 bpm, and respiratory rate is 32/min. On physical examination, the surgical sites are unremarkable. Intravenous fluid resuscitation is promptly initiated. Blood cultures are collected and broad-spectrum antibiotics are administered. Which of the following is the most appropriate next step in management?

A) Administering additional fluid resuscitation with hydroxyethyl starch
B) Initiating low-dose dopamine for renal protection
C) Measuring lactate level
D) Narrowing antimicrobial therapy based on culture results
E) Placing a pulmonary artery catheter

A

The correct response is Option C.

Measuring lactate level is the most appropriate next step in management of this patient who is developing sepsis/septic shock in the postoperative period.

In the 2018 update of the 2016 Surviving Sepsis Campaign guidelines, an Hour-1 Bundle was modified to reflect the need to begin resuscitation and management of patients with sepsis and septic shock immediately upon presentation. The Hour-1 bundle includes:

  • Measuring lactate level. To be re-measured in 2 to 4 hours if initial lactate is > 2 mmol/L
  • Obtaining blood cultures prior to administration of antibiotics
  • Administering broad-spectrum antibiotics
  • Rapidly administering 30 mL/kg crystalloid for hypotension or lactate ≥ 4 mmol/L
  • Initiating vasopressors if patient is hypotensive during or after fluid resuscitation to maintain MAP ≥ 65 mmHg

Serum lactate levels, although not a direct measure, can serve as a surrogate of tissue hypoperfusion. Randomized controlled trials have demonstrated a significant reduction in mortality when lactate-guided resuscitation is employed.

The 2016 Surviving Sepsis Campaign recommended against using low-dose dopamine for renal protection (lack of evidence supporting its efficacy), against providing fluid resuscitation with hydroxyethyl starch (higher risk for death compared with other fluids), and against routine use of pulmonary artery catheters (lack of evidence in improving septic patient outcomes). It does recommend narrowing of antimicrobial therapy based on culture results, but these would not be available in this patient’s early phase of management.

2022

112
Q

A 43-year-old woman is scheduled to undergo bilateral tissue expander-based breast reconstruction. Preoperative pectoral nerve 1 and 2 blocks with liposomal bupivacaine with ultrasound guidance is performed. An image is shown. When performing pectoral nerve 1 block in this patient, which of the following is the most appropriate location for infiltration of the local anesthetic?

A) Between the breast parenchyma and pectoralis major
B) Between the pectoralis major and pectoralis minor
C) Between the pectoralis minor and the serratus anterior
D) Between the serratus anterior and the latissimus dorsi

A

The correct response is Option B.

The pectoral nerve (PECS) 1 and 2 anesthetic blocks have become increasingly common methods for regional anesthesia to decrease postoperative pain in many early recovery after surgery (ERAS) protocols. The PECS blocks are thoracic regional fascial plane blocks that typically require ultrasound guidance for proper infiltration of the anesthetic between the muscles of the thoracic wall.

A PECS 1 block is performed between the pectoralis major and pectoralis minor muscles and anesthetizes the lateral and medial pectoral nerves. A PECS 2 block, in addition to the PECS 1 block, involves an additional injection of anesthetic between the pectoralis minor and serratus anterior muscles and blocks the intercostal and intercostobrachial nerves. The serratus plane block is another thoracic regional fascial plane block requiring injection of local anesthetic between the serratus anterior and latissimus dorsi muscles in order to block the long thoracic and thoracodorsal nerves.

2022

113
Q

A 32-year-old woman presents for bilateral reduction mammaplasty. She is administered 1 g tranexamic acid (TXA) intravenously before incision. Which of the following best describes the mechanism of action of TXA?

A) Activates proteins C and S
B) Inhibits antithrombin III
C) Inhibits platelet aggregation
D) Inhibits proteolysis of protease-activated receptor-1
E) Prevents plasmin from binding to fibrin

A

The correct response is Option E.

Tranexamic acid is a synthetic reversible competitive inhibitor to the lysine receptor found on plasminogen. The binding of this receptor prevents plasmin (the activated form of plasminogen) from binding to and ultimately stabilizing the fibrin matrix. Heparin inhibits antithrombin. Aprotinin is a competitive inhibitor of several serine proteases, specifically trypsin, chymotrypsin, and plasmin, by inhibiting proteolysis of protease-activated receptor-1. Inhibition of platelet aggregation and activation of protein C and S are antithrombotic mechanisms.

2023

114
Q

A 25-year-old woman who weighs 143 lb (65 kg) undergoes awake liposuction in an accredited office-based procedural room with a registered nurse and a surgeon. She was preoperatively treated with oral alprazolam 1 mg and intramuscular meperidine 50 mg. Intraoperatively, the patient is given tumescent lidocaine solution to 45 mg/kg. During the procedure, she reports that she is experiencing a metallic taste in her mouth and has a numbing sensation in her lips. Fifteen minutes later, she exhibits abrupt signs of confusion and agitation; her blood pressure has fallen from 125/88 mmHg to 95/60 mmHg, and her heart rate has increased from 80 bpm to 108 bpm. Which of the following is the most appropriate initial management of this patient?

A) Administer flumazenil 0.2 mg intravenously over 15 seconds
B) Administer an intramuscular injection of naloxone hydrochloride to the thigh
C) Administer 20% lipid emulsion bolus 1.5 mL/kg intravenously over 1 minute
D) Secure the airway to maintain oxygenation and prevent hypoxia and acidosis
E) Stop the procedure and call for help

A

The correct response is Option E.

Local anesthetic systemic toxicity can result in serious patient harm, including seizure, cardiac compromise, and fatality. Local anesthetics act on all parts of the nervous system and all nerve types. The primary therapeutic effect of local anesthetics is to block the action potential generated and propagated in peripheral nerves. Ester local anesthetics are metabolized in the circulation by plasma esterases, whereas amide local anesthetics (ie, lidocaine, mepivacaine, bupivacaine, ropivacaine) are processed in the liver by cytochrome P450-linked enzymes.

The canonical molecular structure of local anesthetics contains a lipophilic aromatic ring system attached by an ester or amide intermediate chain to a hydrophilic tertiary amine. At physiological pH, local anesthetics exist as a mixture of a protonated acid and neutral base because of their tertiary amine. However, only the uncharged form can cross the membrane, and once intracellular, the ionized cationic form binds to a site on the inner pore of the sodium channel. Lowering the arterial pH may create intracellular ion trapping, thereby modifying cellular effects and potentially worsening toxicity.

Local anesthetic systemic toxicity can result from either inadvertent intravascular injection or delayed absorption into circulation from the site of injection or application. Large volumes of local anesthetics are used for cosmetic procedures, including tumescent liposuction. Klein and Jeske have been leaders in the study of safe tumescent anesthesia and suggested that infiltration of lidocaine doses up to 45 mL/kg is safe. However, individual patient sensitivities, comorbidities, and physician or practice deficiencies may predispose to catastrophic outcomes. Furthermore, there are no universal mandated or regulatory reporting requirements that would reliably capture all catastrophic local anesthetic systemic toxicity events. Reporting of fatalities attributable to lidocaine toxicity in otherwise healthy patients following tumescent liposuction has occurred. Cardiac arrest and death caused by local anesthetic systemic toxicity have occurred at outpatient plastic surgery centers, possibly related to large volumes or improper administration.

Management of serious local anesthetic systemic toxicity is different from other cardiac arrest scenarios. For local anesthetic systemic toxicity, airway maintenance and oxygenation are the priority in treatment, because both hypoxia and acidosis exacerbate clinical local anesthetic systemic toxicity. Drug administration in response to cardiac arrest caused by local anesthetic systemic toxicity is also different because the standard adult dose of epinephrine (1 mg) can impair resuscitation from local anesthetic systemic toxicity. Therefore, in this scenario, the individual doses of epinephrine should be decreased to less than 1 μg/kg. Pharmacologic management of local anesthetic systemic toxicity should avoid use of vasopressin, calcium channel blockers, beta-adrenergic blockers, or other local anesthetics. A practitioner who suspects local anesthetic systemic toxicity should immediately stop the procedure and call for assistance.

In a local anesthetic toxicity event, a summary of the critical steps required to manage the event after stopping the procedure includes: airway management (oxygenate and ventilate), secure intravenous access, seizure suppression, and, if needed, cardiopulmonary resuscitation. Alert the nearest facility that has a cardiopulmonary bypass capability, and administer 20% lipid emulsion.

Flumazenil is used to reverse the effects of benzodiazepine toxicity. Naloxone hydrochloride is used to reverse the effect of opiates.

2023

115
Q

A 70-year-old woman is evaluated in the office for delayed breast reconstruction. The modified 5-Item Frailty Index is used to estimate her perioperative risk. A history of which of the following is considered in the calculation of this index?

A) Cerebrovascular accident
B) Congestive heart failure
C) Coronary artery disease
D) Peripheral vascular disease
E) Pulmonary hypertension

A

The correct response is Option B.

While advanced age has been historically considered a risk factor for increased postoperative complications, this idea has been challenged in the literature, particularly in plastic surgery where many procedures are elective and patients can be functionally optimized for surgery. More recently, the concept of frailty has been suggested as an alternative and better predictor for poor surgical outcomes. While the original Canadian Study of Health and Aging Frailty Index consisted of 70 items, the modified 5-Item Frailty Index has shown similar predictive value with increased simplicity. The components of the modified 5-Item Frailty Index include the
following:

  • Functional status before surgery
  • Diabetes mellitus
  • Chronic obstructive pulmonary disease
  • Congestive heart failure
  • Hypertension requiring medication

2023

116
Q

A 55-year-old woman with lipodystrophy after massive weight loss is scheduled to undergo mastopexy and brachioplasty. She underwent Roux-en-Y gastric bypass 10 years ago. Preoperative complete blood count demonstrates macrocytic anemia. This condition is most likely associated with deficiency of which of the following?

A) Iron
B) Vitamin A
C) Vitamin B12 (cobalamin)
D ) Vitamin C
E) Vitamin D
F) Vitamin E

A

The correct response is Option C.

Vitamin B12 (cobalamin) deficiency may cause megaloblastic anemia, a type of macrocytic anemia.

The process of vitamin B12 absorption is started by gastric acidity, which separates this vitamin from the food proteins it is attached to. Next, before it can be absorbed in the terminal ileum, vitamin B12 needs to be combined with a protein secreted by the stomach, called the intrinsic factor.

Bariatric procedures that include partial gastrectomy can substantially decrease the production of hydrochloric acid and intrinsic factor by the stomach, disrupting the absorption of vitamin B12 from one’s diet. Thus, at least annual screening is recommended for patients who have undergone Roux-en-Y gastric bypass. Prevalence of vitamin B12 deficiency in the postbariatric population is thought to be as high as 18%.

Interestingly, serum measurement alone may not be adequate to identify vitamin B12 deficiency. It is recommended to include the serum concentration of methylmalonic acid (MMA) with or without homocysteine to identify metabolic deficiency of B12 in symptomatic and asymptomatic patients, in patients with history of B12 deficiency, or in patients with preexisting neuropathy.

Megaloblastic anemia is characterized by ineffective erythropoiesis resulting from disrupted DNA synthesis, most commonly caused by deficiency of vitamin B12 or folic acid (folate). An incidental finding in routine laboratory testing is the most common presentation, since anemia usually develops gradually and symptoms are present only in severely anemic patients. When caused by vitamin B12 deficiency, neurologic symptoms can be observed, including balance disorder, paresthesias, and lower extremity pain. Diagnostic suspicion should be raised when a complete blood count shows anemia and high mean corpuscular volume or high mean corpuscular hemoglobin.

The other vitamins listed are not associated with macrocytic anemias. Iron deficiency anemia may be common in patients with massive weight loss, but it causes a microcytic anemia.

2023

117
Q

Which of the following methods of achieving osteosynthesis is most likely to result in relative stability?

A) Compression plate
B) Intramedullary nail
C) Lag screws
D) Tension band

A

The correct response is Option B.

Absolute stability is achieved by interfragmentary compression, which allows for direct bone healing without the formation of a callus. Forms of this osteosynthesis include lag screws, compression plates, and tension banding. Relative stability does not have interfragmentary compression at the fracture site, and healing is achieved through callus formation. Types of relative stability include intramedullary nailing, bridge plating, external fixation, and splinting/casting.

2023

118
Q

A 63-year-old woman undergoes sternal reconstruction. On postoperative day 1, she goes into cardiac arrest, advanced cardiac life support protocols are initiated, and pulse resumes. Four days later, the patient remains ventilator-dependent, unresponsive, and in a comatose state. Which of the following physical examination findings provides the strongest support of brain death/death by neurologic criteria in this patient?

A) Absent pupillary response to light
B) Loss of bicep reflexes
C) Positive Babinski reflex
D) Positive Hoffman sign
E) Urinary incontinence

A

The correct response is Option A.

Use of the terms “whole brain death” and “brainstem death” is no longer recommended. Currently, the term brain death/death by neurologic criteria should be used. According to Greer et al, brain death/death by neurologic criteria can be defined as “the complete and permanent loss of brain function as defined by an unresponsive coma with loss of capacity of consciousness, brainstem reflex, and the ability to breathe independently.”

In this scenario, the patient is not conscious and is in a coma, in addition to being unable to breathe independently. Of the listed responses, the pupillary reflex is the only brainstem reflex.

2023

119
Q

A 31-year-old woman is evaluated for augmentation mammaplasty and mastopexy. She sees a local primary care physician who specializes in medical weight loss. In addition to diet modifications and a rigorous exercise regimen, she was also prescribed phentermine. Which of the following is the most common perioperative complication associated with administration of this drug?

A) Cardiac dysrhythmia
B) Hypotension
C) Metabolic acidosis
D) Metabolic alkalosis
E) Seizures

A

The correct response is Option B.

Phentermine is a sympathomimetic amine that is prescribed for weight loss. Phentermine is a centrally acting sympathomimetic that is structurally related to amphetamines. It stimulates the hypothalamus to release norepinephrine, causing appetite suppression. Historically, formulations of the drug included a combination treatment with fenfluramine known as “fen-phen,” which was discontinued in the late 1990s after several patient deaths stemming from cardiorespiratory complications. Over the last decade, however, stand-alone treatment with phentermine has been demonstrated to be safe and effective, and it remains the most prescribed anti-obesity weight loss drug in the country.

Perioperatively, phentermine is associated with refractory hypotension on the induction of general anesthesia, hypothetically stemming from catecholamine depletion and subsequent autonomic dysfunction. In this situation, hypotension may not respond to vasopressors that stimulate catecholamine release (eg, ephedrine), and direct sympathetic agonists (eg, phenylephrine) may be needed.

Other complications described in the literature include hypertension and hyperthermia, but these are much less common. Cardiac dysrhythmia, metabolic acidosis, and metabolic alkalosis are not described complications associated with phentermine use, and neither are seizures in the perioperative period.

2023

120
Q

Which of the following laboratory values is most likely to be abnormal in a patient who takes daily aspirin?

A) Activated partial thromboplastin time
B) Bleeding time
C) D-dimer concentration
D) Platelet count
E) Prothrombin time/International Normalized Ratio

A

The correct response is Option B.

Aspirin causes an increase in bleeding time through the inhibition of prostaglandin biosynthesis and platelet secretion reaction. Platelet function is altered, but not the platelet count. Prothrombin time/International Normalized Ratio, activated partial thromboplastin time, and D-dimer concentration are unaffected by routine aspirin use.

2023

121
Q

A 30-year-old woman undergoes tumescent liposuction with fat transfer to the buttocks (Brazilian butt lift). History includes anxiety, for which she takes sertraline. Twelve hours after the patient is discharged from the recovery room, she presents to the emergency department with nausea, vomiting, and anxiety. Which of the following is the most appropriate next step in management?

A) Administer benzodiazepine
B) Administer supplemental oxygen, and measure plasma lidocaine concentration
C) Obtain blood cultures and complete blood count, and administer broad-spectrum antibiotics
D) Order electrocardiography
E) Order ultrasonography of the lower extremities

A

The correct response is Option B.

This patient is showing signs of lidocaine toxicity. Blood lidocaine concentration peaks approximately 12 hours after tumescent infiltration. Many authors have reported safe use of lidocaine in tumescent liposuction at doses of 35 mg/kg or higher. Toxic effects are generally considered to occur at blood concentrations of 6.0 mg/L (μg/mL) and greater; however, signs of toxicity may occur at lower plasma concentrations. These include gastrointestinal effects (commonly nausea and vomiting), central nervous system effects (such as anxiety, confusion, or light-headedness), and more severe and life-threatening effects (such as respiratory depression, seizures, and cardiovascular collapse). Selective serotonin reuptake inhibitors (such as sertraline) are competitive inhibitors of CYP3A4, which is the same enzyme that metabolizes lidocaine. A patient showing systemic effects of lidocaine toxicity should be treated with oxygen administration, and their plasma lidocaine concentration should be checked. If increased, then overnight observation in the hospital is warranted.

Benzodiazepine treatment is not indicated for anxiety, and in fact, could further depress the respiratory system. It would only be indicated for treatment of lidocaine-induced seizures.

Electrocardiography is not the study of choice for evaluating lidocaine toxicity.

Obtaining blood cultures and complete blood count and administering broad-spectrum antibiotics are the first-line treatment of sepsis, but they are not indicated in this patient.

Ultrasonography of the lower extremities can evaluate for deep venous thrombosis, but it is not indicated in this patient.

2023

122
Q

A 28-year-old man who underwent transplantation of an upper extremity at the mid humeral level 6 hours ago is evaluated in the intensive care unit because his urine output is decreasing. On examination, the patient’s urine is red-amber in color. Electrocardiography (ECG) shows T waves that are almost as tall as the QRS complex on the rhythm strip. In addition to fluid management and obtaining formal 12-lead ECG, the most appropriate next step is to order measurement of which of the following blood electrolytes?

A) Calcium
B) Magnesium
C) Phosphate
D) Potassium
E) Sodium

A

The correct response is Option D.

Replantation of composite tissues with high metabolic tissues such as muscle can result in an inflammatory reperfusion syndrome marked by acidosis, myoglobinuria, and hyperkalemia as the by-products of cellular breakdown are released into the circulatory system. In this scenario, myoglobinuria is suggested by the color of the urine and hyperkalemia becomes suspect with the peaked T waves. Calcium and magnesium can be used as cell membrane stabilizers to mitigate the effect of the potassium on the cardiac muscle. Glucose and insulin are used to move potassium intracellularly. Sodium balance is not directly affected in reperfusion. Phosphate plays a role in bone physiology.

2023

123
Q

A 45-year-old woman is admitted to the hospital because of multiple fluid collections in bilateral gluteal regions. She underwent abdominoplasty and gluteal fat grafting 2 weeks ago while traveling out of the country. She has obstructive sleep apnea, for which she uses a continuous positive airway pressure (CPAP) device at night. She is obese with a BMI of 42 kg/m2 . Vital signs are within normal limits. Surgical exploration of the gluteal collections is planned in the next 48 hours. The use of a CPAP device for this patient’s obstructive sleep apnea is most appropriate during which of the following perioperative periods?

A) Immediate postoperative only
B) Neither preoperative nor immediate postoperative
C) Preoperative and immediate postoperative
D) Preoperative only

A

The correct response is Option C.

This patient with an established diagnosis of obstructive sleep apnea (OSA) should continue to use a continuous positive airway pressure (CPAP) device at night throughout her stay in the hospital, both preoperatively and immediately postoperatively.

Patients with OSA should be considered at an increased risk for perioperative complications, with studies reporting a two- to three-fold increase in adverse cardiopulmonary events.

Sudden discontinuation of CPAP use by patients previously adherent to treatment has been shown to result in recurrence of OSA-related symptoms within 1 to 3 days and physiologic derangements within 2 weeks.

An increasing number of studies suggest that CPAP therapy may play a role in mitigating the increased risk for perioperative complications in patients with OSA. Therefore, continued use of CPAP therapy is recommended during periods of sleep while hospitalized, including before and as soon as feasible after surgery. Adjustments may need to be made to the patient’s baseline CPAP device settings to account for perioperative changes, such as facial swelling, upper airway edema, fluid shifts, pharmacotherapy, and respiratory function.

2023

124
Q

A 35-year-old woman is scheduled to undergo wide local excision of a 3-mm–thick melanoma. General endotracheal anesthesia is induced, and a dual tracing approach is implemented with technetium 99m and isosulfan blue dye. Lidocaine 1% with epinephrine is injected intradermally for local bleeding control. Within moments, the patient becomes hypotensive, develops global urticaria, and desaturates. She progresses to cardiac arrest. Which of the following is the most likely cause of these symptoms?

A) Anaphylaxis to isosulfan blue dye
B) Anaphylaxis to lidocaine
C) Anaphylaxis to skin preparation agent
D) Histamine release from general anesthesia induction with depolarizing agent
E) Malignant hyperthermia

A

The correct response is Option A.

Anaphylaxis to isosulfan blue dye is a well-documented risk with an incidence of 1 to 3%. This patient’s symptoms are not consistent with malignant hyperthermia. Additionally, local anesthesia was used during the punch biopsy, so anaphylaxis to lidocaine is also unlikely. A skin preparation agent anaphylaxis is improbable with the severity of the symptoms. Lastly, a depolarizing agent like succinylcholine can cause histamine release, but the effects are usually transient.

2023

125
Q

A 29-year-old woman who weighs 176 lb (80 kg) undergoes fleur-de-lis abdominoplasty after a weight loss of 65 lb. During abdominal wall plication, 266 mg of liposomal bupivacaine combined with 40 mL of 0.25% bupivacaine hydrochloride is injected along the plication line. During skin closure, 40 mL of 1% lidocaine hydrochloride with 1:100,000 epinephrine is injected along the incisions. Which of the following is most accurate regarding the dose of liposomal bupivacaine in this patient?

A) It is reserved for patients undergoing regional analgesia (eg, brachial plexus blocks)
B) It is reserved for patients weighing over 220 lb (100 kg)
C) It may be safely combined with both 0.25% bupivacaine hydrochloride and 1%
lidocaine hydrochloride
D) It may be safely combined with 0.25% bupivacaine hydrochloride only
E) It may be safely combined with 1% lidocaine only

A

The correct response is Option D.

Conventional local anesthetics provide effective postoperative analgesia but are limited by relatively short durations. Even longer-acting local anesthetics, such as bupivacaine hydrochloride, only last about 8 hours. Liposomal bupivacaine (Exparel) is a novel formulation designed for a rapid absorption and prolonged release of bupivacaine that maintains analgesia for up to 96 hours after surgery.

Liposomal bupivacaine is available in two doses, 266 mg (20 mL) and 133 mg (10 mL). Dosing is not weight-based. The recommended dose is based on the surgical area that requires analgesic coverage. Large areas, such as the abdominal wall in an abdominoplasty, use the larger volume (266 mg), while the lower volume is used for smaller surgical fields, such as the hand, foot, and brachial plexus for regional nerve blocks.

The volume of both doses may be safely expanded with saline or bupivacaine hydrochloride. Twenty mL of liposomal bupivacaine may be expanded with up to 280 mL of normal saline or Ringer’s lactate, or up to 60 mL of 0.25% bupivacaine hydrochloride.

However, while bupivacaine hydrochloride or saline/Ringer’s lactate may be used for volume expansion, the addition of other local anesthetic solutions can alter the release rate of thebupivacaine. Therefore, liposomal bupivacaine and lidocaine should not be mixed together or injected into the same area at the same time. In this patient, injecting liposomal bupivacaine into the abdominal wall plication and lidocaine hydrochloride along the vertical component of a fleur-de-lis incision may cause rapid release of the active medication and alter the safety and efficacy of liposomal bupivacaine. Consequences of bupivacaine toxicity include methemoglobinemia, central nervous system reactions (eg, seizures, respiratory arrest), and cardiovascular dysfunction (eg, hypotension, heart block, dysrhythmias, cardiac arrest).

2023

126
Q

A 42-year-old woman undergoes a circumferential body lift following a 110-lb (49.9-kg) massive weight loss due to a gastric sleeve. Eighteen hours after surgery, she begins vomiting and is noted to have disordered eye movements, ataxia, and mental status changes. Administration of which of the following is most appropriate to treat the symptoms in this patient?

A) Calcium
B) Folate
C) Iron
D) Vitamin B1 (thiamine)
E) Vitamin B12 (cyanocobalamin)

A

The correct response is Option D.

All of the deficiencies listed are common in massive weight loss patients who present for plastic surgery. It is critical to identify that this woman is exhibiting signs of Wernicke encephalopathy (WE). It is characterized by the classic triad of ataxia, eye movement disorders, and mental status change. This acute neuropsychiatric syndrome results from a deficiency of thiamine. The total number of reported post-bariatric WE cases is growing, and it is important to keep this in the differential for plastic surgery patients.

Vomiting is the most frequently described presenting symptom. However, the most profound characteristic of WE identified is ataxia, which often presents itself as gait abnormalities up to the full inability to walk or move. Altered mental status manifests as delirium, confusion, and problems in alertness or cognition. Lastly, eye movement disorders, including nystagmus and ophthalmoplegia, resulting from extraocular muscle weakness, are seen in the majority of cases as well.

Adequate and timely thiamine treatment in patients who have undergone bariatric surgery is required to prevent the development of WE, which is a rare but severe complication.

Calcium deficiency presents more slowly, due to decreased absorption, and results in bone density loss, causing an increased risk for fracture. Folate deficiency also presents slowly secondary to decreased absorption. It results in anemia and can cause abnormal embryogenesis in pregnant women. Iron deficiency also presents slowly from a lack of adequate absorption and clinically causes anemia. Vitamin B12 requires stomach acid for it to be released from dietary protein. With a paucity of gastric tissue to produce that acid, this vitamin is also frequently deficient, resulting in a microcytic anemia.

2023